skin tumor mcq

90
A patient was referred from the oncology service because of tumoral lesions. There is no bone marrow involvement. What is the stage on this patient? 1 IIA 2 IIB 3 IIIA 4 IIIB 5 IV Q/Q(M)-482878 Report a Problem A patient was referred from the oncology service because of tumoral lesions. There is no bone marrow involvement. What is the stage on this patient? 2 IIB This patient has the tumor stage of mycosis fungoides(MF). Patients with tumoral lesions are automatically classified as IIB. IA is the limited patch or plaque disease with less than 10% BSA involved. IB is the generalized patch/plaque stage with more than 10%BSA but less than 80% BSA involved. IIA implies lymph node involvement. IIIA is erythroderma without blood involvement. IIIB is erythroderma with low blood tumor burden. The stage IV of MF is divided in three substages: IVA1(high blood tumor burden-Sezary syndrome), IVA2(very abnormal nodes), IVB(visceral involvement). Q/Q(M)-482878 Report a Problem A young woman presents with a single small, firm, umbilicated papule on the face. Biopsy reveals a well-circumscribed lesion located in the upper dermis. Strands of basaloid cells are seen surrounded by fibrotic or desmoplastic stroma. Horn cysts and foci of sebaceous cells and calcification are also noted. How should this patient and her lesion be treated? 1 Reassurance and no further treatment 2 Close clinical follow-up 3 Topical 5-fluorouracil 4 Cryotherapy 5 Local surgical excision Q/Q(M)-476105 Report a Problem A young woman presents with a single small, firm, umbilicated papule on the face. Biopsy reveals a well-circumscribed lesion located in the upper dermis. Strands of basaloid cells are seen surrounded by fibrotic 1

Upload: drtawheed

Post on 26-Dec-2015

207 views

Category:

Documents


5 download

DESCRIPTION

ETAS MCQ with explanation

TRANSCRIPT

Page 1: Skin Tumor MCQ

A patient was referred from the oncology service because of tumoral lesions. There is no bone marrow involvement. What is the stage on this patient?

1 IIA

2 IIB

3 IIIA

4 IIIB

5 IV

  Q/Q(M)-482878  Report a ProblemA patient was referred from the oncology service because of tumoral lesions. There is no bone marrow involvement. What is the stage on this patient?

2 IIB

This patient has the tumor stage of mycosis fungoides(MF). Patients with tumoral lesions are automatically classified as IIB. IA is the limited patch or plaque disease with less than 10% BSA involved. IB is the generalized patch/plaque stage with more than 10%BSA but less than 80% BSA involved. IIA implies lymph node involvement. IIIA is erythroderma without blood involvement. IIIB is erythroderma with low blood tumor burden. The stage IV of MF is divided in three substages: IVA1(high blood tumor burden-Sezary syndrome), IVA2(very abnormal nodes), IVB(visceral involvement).

  Q/Q(M)-482878  Report a ProblemA young woman presents with a single small, firm, umbilicated papule on the face. Biopsy reveals a well-circumscribed lesion located in the upper dermis. Strands of basaloid cells are seen surrounded by fibrotic or desmoplastic stroma. Horn cysts and foci of sebaceous cells and calcification are also noted. How should this patient and her lesion be treated?

1 Reassurance and no further treatment

2 Close clinical follow-up

3 Topical 5-fluorouracil

4 Cryotherapy

5 Local surgical excision

  Q/Q(M)-476105  Report a Problem

A young woman presents with a single small, firm, umbilicated papule on the face. Biopsy reveals a well-circumscribed lesion located in the upper dermis. Strands of basaloid cells are seen surrounded by fibrotic or desmoplastic stroma. Horn cysts and foci of sebaceous cells and calcification are also noted. How should this patient and her lesion be treated?

5 Local surgical excision

This patient has a desmoplastic trichoepithelioma. Desmoplastic trichoepithelioma is a variant of trichoepithelioma, an uncommon adnexal tumor with differentiation toward hair structures. It presents commonly in young women as a small, firm, umbilicated papule on the face. Local surgical excision is the treatment of choice.

  Q/Q(M)-476105  Report a Problem

Dermoscopic features suggestive of malignant melanoma include:

1 Presence of 2 or more colors within the lesion

2 Blue-whitish veil

3 Asymmetric radial streaming

4 Abrupt interruption of pigment network in the periphery

1

Page 2: Skin Tumor MCQ

5 All of these answers are correct

  Q/Q(M)-474426  Report a Problem

Dermoscopic features suggestive of malignant melanoma include:

5 All of these answers are correct

Asymmetry, multicomponent pattern, blue-whitish veil, parallel-ridge pattern, atypical pigment network, uneven radial streaming, localized irregular and diffuse pigmentation, irregularly distributed globules, and regression structures are all dermoscopic features suggestive of malignant melanoma.

  Q/Q(M)-474426  Report a Problem

All of the following are true of thick melanomas (>3 mm) except:

1 Predominantly nodular type

2 Women affected more than men

3 Predilection for the head and neck

4 Mainly in older patients (>50 years)

5 Associated with fewer nevi

  Q/Q(M)-477153  Report a Problem

All of the following are true of thick melanomas (>3 mm) except:

2 Women affected more than men

According to a study performed by Chamberlain, et.al., thick melanomas (> 3 mm) were predominantly nodular in type. They occurred in older men, mostly on the head and neck and were associated with fewer nevi.

  Q/Q(M)-477153  Report a Problem

Which syndrome is characterized by multiple keratoacanthomas beginning in childhood?

1 Muir-Torre

2 Ferguson Smith

3 Rombo

4 Grybowski

5 Keratoacanthoma centrifugum marginatum

  Q/Q(M)-482675  Report a Problem

Which syndrome is characterized by multiple keratoacanthomas beginning in childhood?

2 Ferguson Smith

Ferguson Smith is familial (AD), and is characterized by regressing keratoacanthomas beginning in childhood. Grybowski is a non-familial disorder in which patients develop generalized eruptive keratoacanthomas beginning between age 40 to 60. Keratoacanthoma centrifugum marginatum is most commonly a solitary lesion occurring later in life. Muir Torre is a syndrome characterized by gastrointestinal or genitourinary neoplasms which precede development of sebaceous neoplasms and keratoacanthomas by one to two decades. Rombo is not associated with keratoacanthomas, but is associated with multiple basal cell carcinomas, atrophoderma vermiculatum, hypotrichosis, and milia.

  Q/Q(M)-482675  Report a ProblemA patient presents with tender papules with a pseudo-Darier's sign. She has other family members with the same condition. Screening should be performed to rule out which malignancy?

1 Gastric carcinoma

2 Ovarian carcinoma

2

Page 3: Skin Tumor MCQ

3 Renal cancer

4 Testicular cancer

5 Lung cancer

  Q/Q(M)-476648  Report a Problem

A patient presents with tender papules with a pseudo-Darier's sign. She has other family members with the same condition. Screening should be performed to rule out which malignancy?

3 Renal cancer

Reed's syndrome is an autosomal dominant disease with incomplete penetrance characterized by uterine and cutaneous leiomyomas. Clinically, leiomyomas appear as flesh colored or pinkish-brown dermal papules or nodules that range from 0.2 to 2.0 cm in diameter. Their presentation may otherwise be quite variable. They may be isolated or many in number, may be variably distributed or dermatomal, and may be asymptomatic or painful in response to pressure or cold. The predisposition gene for Reed�s syndrome has been localized to chromosome 1q42.3-43 and the gene encoding fumarate hydratase. Currently, the United States National cancer Institute is recommending screening for all patients with leiomyomatosis to evaluate for an occult renal malignancy.

  Q/Q(M)-476648  Report a ProblemWhich of the following ethnic groups are commonly diagnosed with dermatosis papulosa nigra:

1 Asians

2 Hispanics

3 African-Americans & Hispanic patients

4 Caucasians

5 No difference between ethnic groups

  Q/Q(M)-474394  Report a Problem

Which of the following ethnic groups are commonly diagnosed with dermatosis papulosa nigra:

3 African-Americans & Hispanic patients

Characterized by the presence of multiple, small, hyperpigmented, sessile SKs on the face, DPN is typically diagnosed on the African-American and Hispanic population.

  Q/Q(M)-474394  Report a Problem

Which of the following is an immunhistochemical marker for Merkel Cell Carcinoma?:

1 S-100

2 Vimentin

3 HMB-45

4 Neuron specific enolase

5 All of these answers are correct

  Q/Q(M)-474400  Report a Problem

Which of the following is an immunhistochemical marker for Merkel Cell Carcinoma?:

4 Neuron specific enolase

Neuron specific enolase stains merkel cells. Vimentin stains melanocytic lesions, sarcomas and lymphomas. S-100 and HMB-45 stains melanocytic lesions, such as melanoma.

  Q/Q(M)-474400  Report a Problem

Which of the following is a common location of melanoma in women?

3

Page 4: Skin Tumor MCQ

1 Chest

2 Lower legs

3 Genitals

4 Digits

5 Scalp

  Q/Q(M)-474446  Report a ProblemWhich of the following is a common location of melanoma in women?

2 Lower legs

The most common locations of MM in women are the back, lower legs and upper extremities.  Q/Q(M)-474446  Report a Problem

A patient has a malignant melanoma 1.6mm thick with ulceration and a micrometastasis in 1 node. The patient's staging according to the American Joint Committee on Cancer Staging System is:

1 IIC

2 IIIA

3 IIIB

4 IIIC

5 IV

  Q/Q(M)-476101  Report a Problem

A patient has a malignant melanoma 1.6mm thick with ulceration and a micrometastasis in 1 node. The patient's staging according to the American Joint Committee on Cancer Staging System is:

3 IIIB

An ulcerated tumor of any size with micrometastasis in 1 node is T1-4b N1a M0. This corresponds to stage IIIB.

  Q/Q(M)-476101  Report a ProblemCompared with the general population, what is the overall risk of developing cutaneous and systemic malignancies in organ transplant recipient?

1 4 fold

2 10 fold

3 25 fold

4 50 fold

5 100 fold

  Q/Q(M)-477383  Report a Problem

Compared with the general population, what is the overall risk of developing cutaneous and systemic malignancies in organ transplant recipient?

1 4 fold

Organ transplant recipients are at increased risk of having both systemic and cutaneous develop. The estimated increased risk is 3-4 times that of the general population. However, the risk of skin cancer alone is much higher (e.g. SCC � 65x increase, BCC 10-fold increase, Melanoma 3.4x increase). This increased risk is thought to be partially due to the immunosuppressant agents used to prevent graft rejection.

  Q/Q(M)-477383  Report a Problem

A 30 year-old gentleman presents with multiple self healing lesions similar to the one pictured. Other family families also have the same disease. What is the most likely diagnosis?

4

Page 5: Skin Tumor MCQ

1 Epidermodysplasia verruciformis

2 Ferguson-Smith syndrome

3 Nevoid basal cell carcinoma syndrome

4 Dyskeratosis congenital

5 Basex syndrome

  Q/Q(M)-476838  Report a Problem

A 30 year-old gentleman presents with multiple self healing lesions similar to the one pictured. Other family families also have the same disease. What is the most likely diagnosis?

2 Ferguson-Smith syndrome

Ferguson-Smith syndrome is a rare autosomal dominant condition that is characerized by multiple keratoacanthomas found in sun-exposed areas. Lesions typically regress over weeks to months.

  Q/Q(M)-476838  Report a Problem

The most common location of superficial spreading melanoma in men is:

1 Lower legs

2 Back

3 Upper extremities

4 Head and neck

5 Digits

  Q/Q(M)-474444  Report a Problem

The most common location of superficial spreading melanoma in men is:

2 Back

Superficial spreading melanoma may be located anywhere, but the back is the most common site in male patients.

  Q/Q(M)-474444  Report a Problem

Imiquimod is an immune response modifier that stimulates innate and cell mediated immune pathways. It induces all of the following cytokines EXCEPT:

1 IL-1

2 IL-4

3 IL-5

4 IL-6

5 IL-8

5

Page 6: Skin Tumor MCQ

  Q/Q(M)-476109  Report a Problem

Imiquimod is an immune response modifier that stimulates innate and cell mediated immune pathways. It induces all of the following cytokines EXCEPT:

2 IL-4

Imiquimod induces the synthesis and release of cytokines such as interleukins 1, 5, 6, 8, 10, and 12, among others.

  Q/Q(M)-476109  Report a Problem

What kind of T-lymphocyte is the most common neoplastic cell in Cutaneous T-Cell Lymphoma:

1 CD4

2 CD8

3 Natural killer

4 None of these answers are correct

5 All of these answers are correct

  Q/Q(M)-474420  Report a Problem

What kind of T-lymphocyte is the most common neoplastic cell in Cutaneous T-Cell Lymphoma:

1 CD4

CTCL is a neoplasm of helper T cells (CD4) that originates in the skin.  Q/Q(M)-474420  Report a Problem

The risk of metastasis from SCC increases with:

1 Tumor size

2 Depth of invasion

3 Degree of differentiation

4 Immunosupression

5 All of these answers are correct

  Q/Q(M)-474419  Report a ProblemThe risk of metastasis from SCC increases with:

5 All of these answers are correct

Tumor size (greater than 2 cms), location (lips, ears, eyelids), depth of invasion, degree of differentiation, perineural invasion, immunosupression, as well as recurrent tumors and those that arise in areas of chronic inflammation are all factors that increase the risk of metastasis.

  Q/Q(M)-474419  Report a Problem

All of the following statements regarding Basal Cell Nevus Syndrome are true EXCEPT:

1 Associated tumors include medulloblastoma

2 It is inherited in an autosomal recessive manner

3 Hypertelorism is a feature

4 The affected gene is mutated in 30-40% of sporadic basal cell carcinomas

5 All of these answers are correct

  Q/Q(M)-476111  Report a Problem

All of the following statements regarding Basal Cell Nevus Syndrome are true EXCEPT:

6

Page 7: Skin Tumor MCQ

2 It is inherited in an autosomal recessive manner

Basal cell nevus syndrome (Gorlin Syndrome) is caused by a mutation in the PTCH gene, located on chromosome 9q22. This gene is found to be mutated in 30-40% of sporadic basal cell carcinomas. Gorlin Syndrome is inherited in an autosomal dominant pattern. Tumors associated with this sydrome include medulloblastoma and meningioma. It is characterized by the appearance of multiple BCCs during childhood, odontogenic keratocysts of the jaw, and skeletal defects (including frontoparietal bossing and hypertelorism, among others).

  Q/Q(M)-476111  Report a Problem

Which of the following features of thin melanomas (<1 mm thick) has been associated with an increased risk of metastasis?

1 Regression

2 Location

3 Size of lesion

4 Gender

5 P53 expression

  Q/Q(M)-477138  Report a Problem

Which of the following features of thin melanomas (<1 mm thick) has been associated with an increased risk of metastasis?

1 Regression

Regression in malignant melanoma appears histologically as a focal area of fibrosis with lymphocytes and melanophages in the papillary dermis. The presence of regression precludes accurate measurement of true thickness of the melanoma. Extensive regression in thin melanomas has a significant association for the risk of metastases.

  Q/Q(M)-477138  Report a ProblemPseudorosettes in Merkel cell carcinoma:

1 Are seen in the trabecular variant

2 Are seen in the intermediate-cell type

3 Are seen in the small-cell type

4 All of these answers are correct

5 None of these answers are correct

  Q/Q(M)-476103  Report a Problem

Pseudorosettes in Merkel cell carcinoma:

1 Are seen in the trabecular variant

There are three histologic patterns of MCC: trabecular, intermediate-cell type, and small-cell type. The trabecular variant consists of interconnecting trabeculae separated by strands of connective tissue. Pseudorosettes may be seen in this type.

  Q/Q(M)-476103  Report a Problem

UVB induced mutations on the PTCH gene is associated with the development of:

1 BCC

2 Merkel cell carcinoma

3 Angiosarcoma

4 BCC and Merkel cell carcinoma

7

Page 8: Skin Tumor MCQ

5 Merkel cell carcinoma and Angiosarcoma

  Q/Q(M)-474402  Report a Problem

UVB induced mutations on the PTCH gene is associated with the development of:

1 BCC

The p53 and PTCH genes are the major targets of UVB for the development of BCC. Other genes involved include the Smoothened-activating mutations, and PTCH2 mutations.

  Q/Q(M)-474402  Report a Problem

Which of the following is the most common initial site of metastasis from a primary BCC?

1 Lungs

2 Regional lymph nodes

3 Bone

4 Liver

5 Pleura

  Q/Q(M)-474439  Report a Problem

Which of the following is the most common initial site of metastasis from a primary BCC?

2 Regional lymph nodes

The metastatic potential of BCC is very low with rates ranging from 0.0028 to 0.1%. The head and neck region is the most frequent location of the primary tumor with regional lymph nodes being the most common site of metastasis. The lungs, bone, liver, and pleural are also potential sites of metastasis.

  Q/Q(M)-474439  Report a Problem

What temperature must be achieved for adequate treatment of a small superficial squamous cell carcinoma with cryotherapy?

1 -10 degrees Celcius

2 -20 degrees Celcius

3 -30 degrees Celcius

4 -40 degrees Celcius

5 -50 degrees Celcius

  Q/Q(M)-479616  Report a Problem

What temperature must be achieved for adequate treatment of a small superficial squamous cell carcinoma with cryotherapy?

5 -50 degrees Celcius

Cryosurgery destroys tumor if it is frozen to -40 to -70 degrees Celcius, for at least 2 cycles with a 60 second thaw. Benign lesions can be destroyed at temperatures around -25 degrees Celcius.

  Q/Q(M)-479616  Report a ProblemThe most common locations of microcystic adnexal carcinoma include all of the following, except:

1 Perioral

2 Nasolabial

3 Trunk

4 Periorbital

5 Perioral,nasolabial, and periorbital

8

Page 9: Skin Tumor MCQ

  Q/Q(M)-474987  Report a Problem

The most common locations of microcystic adnexal carcinoma include all of the following, except:

3 Trunk

Microcystic adnexal carcinomas are more commonly located in the perioral, nasolabial, or periorbital areas.

  Q/Q(M)-474987  Report a Problem

Which of the following are features of microcystic adnexal carcinoma that help distinguish it from desmoplastic trichoepitheliomas?

1 Deep subcutaneous infiltration

2 Perineural invasion

3 CEA positive staining

4 Commonly located on the face

5 Deep subcutaneous infiltration,perineural invasion, and CEA positive staining

  Q/Q(M)-474988  Report a Problem

Which of the following are features of microcystic adnexal carcinoma that help distinguish it from desmoplastic trichoepitheliomas?

5 Deep subcutaneous infiltration,perineural invasion, and CEA positive staining

Desmoplastic trichoepitheliomas is one of the differential diagnosis for MAC. MAC show deep subcutaneous and perineural invasion, as well as CEA positive staining, all features that may help differentiate it from desmoplastic trichoepitheliomas. Both, MAC and desmoplastic trichoepitheliomas are commonly located on the face.

  Q/Q(M)-474988  Report a Problem

The following lesion is the classic presentation of:

1 BCC

2 Merkel cell carcinoma

3 Melanoma

4 CTCL

5 Keratoacanthoma

  Q/Q(M)-474902  Report a Problem

The following lesion is the classic presentation of:

5 Keratoacanthoma

Keratoacanthomas present as a solitary, firm, dome-shaped papule with a cratiform center.  Q/Q(M)-474902  Report a Problem

9

Page 10: Skin Tumor MCQ

Which one of the following malignancies is associated with HPV infection?

1 Verrucous carcinoma

2 Metastatic melanoma

3 Basal cell carcinoma

4 Sebaceous carcinoma

5 Atypical fibroxanthoma

  Q/Q(M)-477339  Report a Problem

Which one of the following malignancies is associated with HPV infection?

1 Verrucous carcinoma

Verrucous carcinomas are low-grade carcinomas which are slow-growing and metastasize very late in the course. The presence of HPV has been demonstrated in cases both by electron microscopy and DNA hybridization.

  Q/Q(M)-477339  Report a Problem

In which of the following ethnic groups is squamous cell carcinoma the most common type of skin cancer?

1 Asian Indians

2 Caucasians

3 Hispanics

4 Japanese

5 Chinese

  Q/Q(M)-482498  Report a Problem

In which of the following ethnic groups is squamous cell carcinoma the most common type of skin cancer?

1 Asian Indians

SCC is the most common type of skin cancer in Asian Indians and Blacks. BCC is the most common type of skin cancer in Caucasians, Japanese, Chinese, and Hispanic people.

  Q/Q(M)-482498  Report a Problem

Which of the following immunohistochemistry marker is negative in angiosarcomas?

1 CD31

2 CEA

3 Cytokeratin

4 CD34

5 Factor VIII

  Q/Q(M)-474424  Report a ProblemWhich of the following immunohistochemistry marker is negative in angiosarcomas?

2 CEA

Angiosarcomas are CD31, CD34, factor VIII, and cytokeratin positive. The carcinoembryonic antigen (CEA) stains positive in Paget�s disease, metastatic adenocarcinoma, and tumors with eccrine differentiation.

  Q/Q(M)-474424  Report a ProblemWhat is the most common site of metastasis for this dermal tumor?

10

Page 11: Skin Tumor MCQ

1 Lung

2 Brain

3 Kidney

4 Liver

5 Bone

  Q/Q(M)-476843  Report a Problem

What is the most common site of metastasis for this dermal tumor?

1 Lung

Dermatofibrosarcoma protuberans is a rare, low-grade dermal sarcoma. Typically, the lesion occurs as a painless subcutaneous mass that grows slowly. This malignancy typically has lateral spread but invade deep. Metastasis is rare but has been reported to the lung.

  Q/Q(M)-476843  Report a Problem

All of the following cytokines have demonstrated therapeutic benefit in the treatment of melanoma EXCEPT:

1 IFN-alpha

2 IL-2

3 TNF-alpha

4 IL-10

5 GM-CSF

  Q/Q(M)-477183  Report a Problem

All of the following cytokines have demonstrated therapeutic benefit in the treatment of melanoma EXCEPT:

5 GM-CSF

IFN-alpha, IL-2, TNF-alpha, and IL-10 have all been demonstrated to have some therapeutic benefit in the treatment of melanoma. IL-10 has been used to treat inflammatory disorders such as atopic dermatitis and psoriasis.

  Q/Q(M)-477183  Report a Problem

Which one of the following agents has demonstrated potential benefit as a chemopreventive to UV-induced skin cancer?

1 Prostaglandin E2

2 Vitamin D

3 Arachidonic acid

4 Celecoxib

11

Page 12: Skin Tumor MCQ

5 Vitamin E

  Q/Q(M)-477331  Report a Problem

Which one of the following agents has demonstrated potential benefit as a chemopreventive to UV-induced skin cancer?

4 Celecoxib

Cyclooxygenase-1 and -2 and enzymes that catalyze the conversion of arachidonic acid to prostaglandins. It is believed the prostaglandin E2 (PGE2), whose levels are increased by ultraviolet irradiation, is pro-inflammatory and may contribute to skin carcinogenesis. In a study by Orengo et.al., hairless mice who were given celecoxib were found to have a significantly longer latency period between exposure to ultraviolet light and the development of skin carcinomas.

  Q/Q(M)-477331  Report a Problem

Intermittent sun exposure with painful sunburns is a predisposing factor for the development of:

1 Atypical nevi

2 Seborrheic keratosis

3 Malignant melanoma

4 Atypical nevi and Malignant melanoma

5 All of these answers are correct

  Q/Q(M)-474429  Report a Problem

Intermittent sun exposure with painful sunburns is a predisposing factor for the development of:

4 Atypical nevi and Malignant melanoma

It has been reported that the risk for the development of atypical nevi and melanoma is higher than twofold with a history of five or more episodes of painful sunburn during adolescence.

  Q/Q(M)-474429  Report a Problem

The incidence of cutaneous squamous cell carcinoma in organ transplant recipients is increased by how much compared with the general population?

1 2 fold

2 5 fold

3 10 fold

4 20 fold

5 65 fold

  Q/Q(M)-477368  Report a Problem The incidence of cutaneous squamous cell carcinoma in organ transplant recipients is increased by how much compared with the general population?

5 65 fold

The risk of skin cancer in organ transplant patients is dramatically increase and may be more aggressive. In one study, SCC�s had a 65x increased incidence, BCC 10x, and melanoma 3.4x.

  Q/Q(M)-477368  Report a ProblemWhich of the following neoplasms has demonstrated an association with HTLV-1 infection?

1 Mycosis fungoides

2 Adult T cell lymphoma

3 Follicular lymphoma

12

Page 13: Skin Tumor MCQ

4 Multiple myeloma

5 Hodgkin�s disease

  Q/Q(M)-477137  Report a Problem

Which of the following neoplasms has demonstrated an association with HTLV-1 infection?

2 Adult T cell lymphoma

Mycosis fungoides is a rare form of cutaneous T-cell lymphoma, the etiology of which is not completely known. Adult T Cell Lymphoma is also a type of T cell neoplasm that has been linked to HTLV-1. Adult T-cell lymphoma may have an acute and chronic, smoldering form. The chronic, smoldering form can be difficult to distinguish from mycosis fungoides.

  Q/Q(M)-477137  Report a Problem

All of the following statements are true regarding angiosarcomas EXCEPT:

1 They occur more commonly in Caucasians than in non-Caucasians

2 Men are more often affected than women

3 They are rarely symptomatic

4 Radiation is usually employed after surgical excision

5 Cervical lymph nodes are a common site of metastases

  Q/Q(M)-476124  Report a Problem

All of the following statements are true regarding angiosarcomas EXCEPT:

3 They are rarely symptomatic

Angiosarcomas are very rare, aggressive vascular tumors. They occur most commonly in the head and neck region of white, elderly individuals. Men are more commonly diagnosed with this neoplasm. The lesion initially arises as a painless, purple macule or plaque on the scalp or face. Later on it becomes an elevated bluish or purple nodule that may ulcerate. Common symptoms include bleeding, edema, and ultimately pain. Cervical lymph node and hematogenous metastases commonly occur. Wide surgical excision is the treatment of choice, with radiation therapy usually employed after surgical excision.

  Q/Q(M)-476124  Report a Problem

Which neoplasm is associated with the Stewart-Treves syndrome?

1 Renal leiomyomas

2 Basal cell carcinoma

3 Angiosarcoma

4 Keratoacanthoma

5 T cell lymphoma

  Q/Q(M)-477129  Report a Problem

Which neoplasm is associated with the Stewart-Treves syndrome?

3 Angiosarcoma

Stewart-Treves syndrome is the development of angiosarcoma in the setting of chronic lymphedema. Originally, named after radical mastectomy for the treatment of breast cancer. The term applies to the development of angiosarcoma in any chronic lymphedematous condition.

  Q/Q(M)-477129  Report a ProblemWhich test should be used to detect monoclonal gene rearrangements in cutaneous T-cell lymphoma?

1 Northern blot

13

Page 14: Skin Tumor MCQ

2 Southern blot

3 Western blot

4 ELISA

5 Electrophoresis

  Q/Q(M)-476531  Report a Problem

Which test should be used to detect monoclonal gene rearrangements in cutaneous T-cell lymphoma?

2 Southern blot

Monoclonality and gene rearrangements can be detected with a Southern blot. Southern blots can be used to detect specific DNA fragments by gel-transfer hybridization.

  Q/Q(M)-476531  Report a Problem

The treatment of choice for this lesion shown is:

1 Radiation therapy

2 Imiquimod

3 5 Flourouracil

4 Wide excision with 2cm margins

5 Mohs surgery

  Q/Q(M)-478126  Report a Problem

The treatment of choice for this lesion shown is:

5 Mohs surgery

The treatment of choice for Dermaotfibrosarcoma protuberans is Mohs surgery. Radiation therapy has been used, however has limited value as solitary therapy for thsi tumor. Raditationtherapy cna be used as an adjunct to wide surgical excision. Calssically these tumors should be excised wtih 3cm margins. The recurrence rate associated with these tumors can be 10-20 percent with wide excision with 3 cm margins. With Mohs surgery, the recurrence rate ranges from 0% to 6%. 5FU and Imiquimod are not effective modalitites in treating DFSPs, as it infiltrates deep into the subcutaneous tissue.

  Q/Q(M)-478126  Report a ProblemWhich of the following would you not expect to see under dermoscopy?

1 Maple leaf pattern

2 Arborizing blood vessels

3 Blue-grey ovoid nests

4 Orange crust

5 Milky red globules

  Q/Q(M)-476837  Report a Problem

14

Page 15: Skin Tumor MCQ

Which of the following would you not expect to see under dermoscopy?

5 Milky red globules

Dermoscopy is a useful tool in differentiating a pigmented basal cell carcinoma from melanoma. Basal cell carcinomas may have arborizing blood vessels, maple leaf pattern, blue-grey ovoid nests, and orange crust or ulcer. Milky red globules are sometimes seen in melanoma.

  Q/Q(M)-476837  Report a Problem

A 56-year old woman with a history significant for chronic lymphedema after radical mastectomy twelve years ago presents with this growth on her arm. What is the diagnosis?

1 Angiosarcoma

2 Bacillary angiomatosis

3 Castleman's syndrome

4 Kaposi's sarcoma

5 Metastatic breast carcinoma

  Q/Q(M)-476834  Report a Problem

A 56-year old woman with a history significant for chronic lymphedema after radical mastectomy twelve years ago presents with this growth on her arm. What is the diagnosis?

1 Angiosarcoma

Angiosarcoma may occur in association with chronic lymphedematous states. Stuart-Treves syndrome applies to the specific condition of an angiosarcoma arising on the upper arm after radical mastectomy.

  Q/Q(M)-476834  Report a Problem

A 60 year-old female presents with a well-demarcated, scaly, erythematous plaque on her right shin. The biopsy shows full thickness epidermal atypia with scattered mitotic figures and overlying parakeratosis. Howerver, the basement membrane remains intact. According to the aforementioned information, the diagnosis is:

15

Page 16: Skin Tumor MCQ

1 BCC

2 SCC

3 Bowen's disease

4 Angiosarcoma

5 CTCL

  Q/Q(M)-474903  Report a ProblemA 60 year-old female presents with a well-demarcated, scaly, erythematous plaque on her right shin. The biopsy shows full thickness epidermal atypia with scattered mitotic figures and overlying parakeratosis. Howerver, the basement membrane remains intact. According to the aforementioned information, the diagnosis is:

3 Bowen's disease

Bowen�s disease arising on the lower limbs is frequently found in women; whereas lesions located on the ears and scalp are more common in men. The epidermal dysplasia does not interrupt the basement membrane confirming the diagnosis of SCC in situ.

  Q/Q(M)-474903  Report a Problem

All of the followings can be used for treatemt of this condition except

1 Cryotherapy

2 Topical Imiquimod 5% cream

3 Topical 5-flurouracil

4 Topical retinoids

5 Surgical excision

  Q/Q(M)-482126  Report a Problem

All of the followings can be used for treatemt of this condition except

5 Surgical excision

Attached picture is disseminated superficial actinic porokeratosis (DSAP) which is the most common type of all porokeratosis, with multiple thin papules appearing most commonly on the legs of adult women. Many treatments have been used for this condition which include: cryotherapy, topical 5-fluorouracil (5-FU), topical retinoids, CO2 laser, and dermabrasion. Although surgical excision might be used for treatment of other forms of porokeratosis, it is not advised in this case because of number of lesions and increase risk of scarring. Other forms of porokeratosis are: porokeratosis of Mibelli, punctate porokeratosis, linear, and Porokeratosis palmaris et plantaris disseminata.

  Q/Q(M)-482126  Report a Problem

If left untreated, which of the following is not at risk for malignant transformation?

1 Bowenoid papulosis

2 Cutaneous horn

16

Page 17: Skin Tumor MCQ

3 Actinic cheilitis

4 Leukoplakia

5 Stucco keratosis

  Q/Q(M)-474397  Report a Problem

If left untreated, which of the following is not at risk for malignant transformation?

5 Stucco keratosis

Cutaneous horn is defined as a hypertrophic AK that presents as a conical protuberance arising from an erythematous base. Actinic cheilitis results from the confluence of multiple AKs on the lips. Leukoplakia is a clinical diagnosis and is defined as a white patch in the oral cavity. It is the most common premalignant condition of the oral cavity. Bowenoid papulosis manifests clinially as multiple red-brown warty papules that histologically represent high grade squamous intraepithelial lesions.

  Q/Q(M)-474397  Report a ProblemWhich of the following variants of mycosis fungoides is best diagnosed using a punch biopsy instead of a broad superficial shave biopsy?

1 Woringer-Kolopp pagetoid reticulosis

2 Syringotropic mycosis fungoides

3 Ketron-Goodman pagetoid reticulosis

4 Poikilodermatous mycosis fungoides

5 Sezary syndrome

  Q/Q(M)-482500  Report a Problem

Which of the following variants of mycosis fungoides is best diagnosed using a punch biopsy instead of a broad superficial shave biopsy?

2 Syringotropic mycosis fungoides

Syringotropic and folliculotropic mycosis fungoides (MF) are the variants of MF that should be diagnosed by punch biopsy. Both variants of pagetoid reticulosis, Woringer-Kolopp disease and the disseminated Ketron-Goodman disease, are best diagnosed with a broad shave biopsy. Classic and poikilodermatous MF should also be diagnosed with a shave biopsy. To rule out Sezary syndrome, flow cytometry should be performed.

  Q/Q(M)-482500  Report a Problem

Merkel cell carcinoma stains positively for:

1 Leukocyte common antigen

2 Neuron specific enolase

3 Vimentin

4 S-100

5 None of these answers are correct

  Q/Q(M)-474436  Report a Problem

Merkel cell carcinoma stains positively for:

2 Neuron specific enolase

Merkel cell carcinoma is frequently diagnosed in individuals older than 50 years of age. However, there have been reports of MCC in children and young adults.

  Q/Q(M)-474436  Report a Problem

17

Page 18: Skin Tumor MCQ

A 3-year-old girl presents with multiple small, angulated, firm nodules. There is a positive "tent sign" and biopsy reveals ghost cells and germinative cells. Of the following conditions associated with multiple pilomatricomas, which is thought to be most closely linked?

1 Turner's

2 Myotonic dystrophy

3 Rubenstein-Taybi

4 Sarcoidosis

5 Gardner's syndrome

  Q/Q(M)-482439  Report a Problem

A 3-year-old girl presents with multiple small, angulated, firm nodules. There is a positive "tent sign" and biopsy reveals ghost cells and germinative cells. Of the following conditions associated with multiple pilomatricomas, which is thought to be most closely linked?

2 Myotonic dystrophy

Pilomatricomas are the most common superficial pediatric tumor. The occurrence of multiple pilomatricomas has been most closely associated with the development of myotonic dystrophy(Steinert Disease). The onset of myotonic dystrophy, an autosomal dominant disorder with variable penetrance, can occur before or after the onset of lesions. Cigliano et al. reported myotonic dystrophy in 1 out of 2 patients with multiple pilomatricomas; Julian et al. reported myotonic dystophy in 1 out of 4 patients. The other syndromes associated with multiple pilomatricomas include Rubenstein-Taybi, Turner syndrome, Gardner syndrome, and sarcoidosis.

  Q/Q(M)-482439  Report a Problem

Which of the following immunohistochemical markers may be indicative of metastatic potential for merkel cell carcinoma?

1 Chromogranin

2 Synaptophysin

3 CD44

4 CK20

5 TTF-1

  Q/Q(M)-482512  Report a Problem

Which of the following immunohistochemical markers may be indicative of metastatic potential for merkel cell carcinoma?

3 CD44

Chromogranin and synaptophysin are positive in merkel cell carcinoma along with neuron-specific enolase and EMA. CK20 is also positive in merkel cell carcinoma and exhibits a paranuclear dot staining pattern. According to a 1996 study, CD44 may indicate metastatic potential. TTF-1 is negative in merkel cell carcinoma, which helps distinguish it from small cell lung carcinoma.

  Q/Q(M)-482512  Report a Problem

All of the following disorders may manifest as exfoliative dermatitis except

1 Behchets disease

2 Psoriasis

3 Pemphigus foliaceus

4 Drug reaction

5 Sezary syndrome

18

Page 19: Skin Tumor MCQ

  Q/Q(M)-481908  Report a Problem

All of the following disorders may manifest as exfoliative dermatitis except

1 Behchets disease

The image shows erythrodermic patient with generalized desquamation of skin. Skin biopsy and blood tests the diagnosis of Sezary syndrome. Sezary syndrome is a leukemic variant of mycosis fungoides, is characterized by the triad of pruritic erythroderma, generalized lymphadenopathy, and the presence of Sezary cells more than 1,000 cell/mm3 (abnormal, large hyperconvoluted lymphocytes) in peripheral blood . Other manifestations include scaling and fissuring of palms and soles, alopecia, pruritus, peripheral edema, and nail dystrophy. Exfoliative dermatits can be manifested in many conditions other than Sezary syndrome inculude pemphigus foliaceus, psoriasis and drug reaction.

  Q/Q(M)-481908  Report a Problem

The most common site for intra oral melanoma is?

1 buccal

2 soft palate

3 hard palate

4 gingiva

5 uvula

  Q/Q(M)-482205  Report a Problem

The most common site for intra oral melanoma is?

3 hard palate

Multiple case series have shown that the hard palate (and specifically the anterior hard palate/alveolar arch) is the highest risk location for intra-oral melanoma. Prognosis is generally worse for oral melanoma, which is usually due to delay in diagnosis and presentation with more invasive disease.

  Q/Q(M)-482205  Report a Problem

A 40 year-old female patient presents with the following lesion (see image). The biopsy report shows duct like structures, tadpole structures within a fibrotic stroma. According to the aforementioned information, the most likely diagnosis is:

1 Microcystic adnexal carcinoma

2 Seborrheic keratosis

3 Dermatofibrosarcoma protuberans

4 Merkel cell carcinoma

5 None of these answers are correct

  Q/Q(M)-474991  Report a Problem

19

Page 20: Skin Tumor MCQ

A 40 year-old female patient presents with the following lesion (see image). The biopsy report shows duct like structures, tadpole structures within a fibrotic stroma. According to the aforementioned information, the most likely diagnosis is:

1 Microcystic adnexal carcinoma

One of the most common locations for MAC includes the perioral area. Histologically it presents with poorly demarcated tumor cells invading the dermal and subcutaneous tissue. Islands of basaloid keratinocytes, horn cysts and duct structures are also seen within a desmoplastic stroma.

  Q/Q(M)-474991  Report a Problem

Sezary syndrome:

1 Has characteristic Sezary cells in peripheral blood

2 Is the leukemic variant of Mycosis fungoides

3Is characterized by the triad of pruritic erythroderma, generalized lymphadenopathy, and presence of Sezary cells in peripheral blood

4 Is associated with a poor prognosis

5 All of these answers are correct

  Q/Q(M)-474438  Report a Problem

Sezary syndrome:

5 All of these answers are correct

Sezary syndrome is the leukemic variant of CTCL characterized by the triad of pruritic erythroderma, generalized lymphadenopathy, and presence of Sezary cells in peripheral blood. Sezary cells are abnormal, large hyperconvoluted lymphocytes. The five year survival is estimated to be between 10-20%.

  Q/Q(M)-474438  Report a ProblemAmplification of which of the following genes is associated with Merkel cell carcinoma?

1 L-Myc

2 C-Myc

3 GLI1

4 CDKN2A

5 PTEN

  Q/Q(M)-482403  Report a ProblemAmplification of which of the following genes is associated with Merkel cell carcinoma?

1 L-Myc

The L-Myc gene has been found to be amplified in Merkel cell carcinoma but not in normal skin. C-Myc has been found to be amplified in neuroblastoma. GLI1 is a transcription factor involved in hedgehog signaling and a potential target in basal cell carcinomas. CDKN2A is implicated in familial forms of melanoma that are associated with pancreatic cancer. PTEN is a tumor suppressor gene that is mutated in

20

Page 21: Skin Tumor MCQ

Cowden Disease, Bannayan-Riley-Ruvalcaba Syndrome, and Proteus Syndrome.  Q/Q(M)-482403  Report a Problem

All of the following are risk factors for metastasis from a primary squamous cell carcinoma EXCEPT:

1 Increasing tumor size

2 Location on the ear

3 Recurrent tumor

4 Tumor within ulcer

5 All of these answers are correct

  Q/Q(M)-476123  Report a Problem

All of the following are risk factors for metastasis from a primary squamous cell carcinoma EXCEPT:

5 All of these answers are correct

The risk of metastasis from a primary SCC increases with tumor size (>2cm), location (lips, ear, and eyelid, among others), depth of invasion, immunosuppression, degree of differentiation, and perineural invasion. Recurrent tumors and SCCs arising in areas of chronic inflammation (such as osteomyelitis, burn scars, or ulcers) are also considered high risk for metastases.

  Q/Q(M)-476123  Report a ProblemWhat is the most likely diagnosis?

1 Acquired digital fibrokeratoma

2 Supernumery digit

3 Verruca

4 Infantile digital fibroma

5 Acrochordon

  Q/Q(M)-476808  Report a Problem

What is the most likely diagnosis?

1 Acquired digital fibrokeratoma

The condition shown is an acquired digital fibrokeratoma. It is a pedunculated, acral lesion with collarette which is defined by the lack of nernve twigs and bone on pathologic exam.

  Q/Q(M)-476808  Report a ProblemWhich phase of the cell cycle does p53 regulate?

1 G1

2 G2

21

Page 22: Skin Tumor MCQ

3 S phase

4 Mitosis

5 Meiosis

  Q/Q(M)-476595  Report a Problem

Which phase of the cell cycle does p53 regulate?

1 G1

p53 is a tumor suppressor gene which arrests cell cycle and downregulates BCL-2. Mutations in p53 are associated with Li-Fraumeni syndrome as well as the development of squamous cell carcinomas.

  Q/Q(M)-476595  Report a ProblemWhich area of the face receives the most cumulative exposure to UV radiation?

1 Dorsum of nose

2 Orbital region

3 Base of the nose

4 Chin

5 Central cheek

Q/Q(M)-477225 Report a Problem

Which area of the face receives the most cumulative exposure to UV radiation?

3 Base of the nose

The most frequent sites of basal cell carcinomas in one study was the base of the nose (bordering the nasolabial fold and extending midway up the side of the nose), orbital region, apex of the nose, ear, forehead, temporal region, nasolabial area, and the buccal region (in order). The nose was by far the most frequent site of BCC�s.

Q/Q(M)-477225 Report a Problem

This tumor, also called a Shagreen patch, is characteristic of which of the following genodermatoses?

1 Neurofibromatosis-1

2 Neurofibromatosis-2

3 Tuberous sclerosis

4 NAME syndrome

5 Buschke-Ollendorf

Q/Q(M)-476842 Report a Problem

22

Page 23: Skin Tumor MCQ

This tumor, also called a Shagreen patch, is characteristic of which of the following genodermatoses?

3 Tuberous sclerosis

Tuberous sclerosis a rare genodermatosis characterized by adenoma sebaceum, seizures and mental retardation. Other cutaneous features include hypopigmented macules, periungual fibromas, fibrous plaque of the face, cafe-au-lait macules and connective tissue nevus.

Q/Q(M)-476842 Report a Problem

This lesion is associated with which of the following:

1 HPV 5

2 HPV 11

3 HPV 4

4 HPV 7

5 HPV 1

Q/Q(M)-477683 Report a Problem

This lesion is associated with which of the following:

2 HPV 11

HPV 6 and 11 are most often associated with anogenital warts or condyloma acuminata. They can present as sessile lesions on the skin or peducnulated cauliflower lesions. HPV type 5 is associated with epidermodysplasia verruciformis, HPV 4 is assoicatd with common warts, HPV type 7 is associated with Butcher warts, and HPV type 1

23

Page 24: Skin Tumor MCQ

is associated with common warts.

Q/Q(M)-477683 Report a Problem

All of the following statements are true regarding angiosarcomas EXCEPT:

1 They occur more commonly in Caucasians than in non-Caucasians

2 Men are more often affected than women

3 They are rarely symptomatic

4 Radiation is usually employed after surgical excision

5 Cervical lymph nodes are a common site of metastases

Q/Q(M)-476124 Report a Problem

All of the following statements are true regarding angiosarcomas EXCEPT:

3 They are rarely symptomatic

Angiosarcomas are very rare, aggressive vascular tumors. They occur most commonly in the head and neck region of white, elderly individuals. Men are more commonly diagnosed with this neoplasm. The lesion initially arises as a painless, purple macule or plaque on the scalp or face. Later on it becomes an elevated bluish or purple nodule that may ulcerate. Common symptoms include bleeding, edema, and ultimately pain. Cervical lymph node and hematogenous metastases commonly occur. Wide surgical excision is the treatment of choice, with radiation therapy usually employed after surgical excision.

Q/Q(M)-476124 Report a Problem

When performing a biopsy of a suspected keratoacanthoma:

1 Fusiform incision through the entire KA may be performed

2 It is necessary to biopsy down to subcutaneous fat

3 A full-thickness shave biopsy is acceptable

4 A complete excisional biopsy may be performed

5 All of these answers are correct

Q/Q(M)-476099 Report a Problem

When performing a biopsy of a suspected keratoacanthoma:

5 All of these answers are correct

When considering a keratoacanthoma, it is important to obtain a biopsy of the specimen down to the subcutaneous fat. This can be achieved either by complete excisional biopsy, full-thickness shave biopsy, or

24

Page 25: Skin Tumor MCQ

fusiform incision through the entire KA including its center and sides.

Q/Q(M)-476099 Report a Problem

Currently, the surgical margin for melanomas that measure less than 2 mm in thickness is:

1 1 mm

2 0.5 cm

3 1 cm

4 2 cms

5 3 cms

Q/Q(M)-474422 Report a Problem

Currently, the surgical margin for melanomas that measure less than 2 mm in thickness is:

3 1 cm

The current surgical margins are 0.5 cms for melanoma in situ, 1 cm for melanomas that measure =2mm in thickness.

Q/Q(M)-474422 Report a Problem

Which of the following melanoma scenarios have the best prognosis?

1 Twenty-one year old female with primary lesion located on the right lower leg

2 Twenty-one year old male with primary lesion located on the chest

3 Thirty-six year old male with primary lesion located on the back

4 Twenty-one year old female with primary, ulcerated lesion located on the right lower leg

5 Thirty-six year old male with primary lesion located on his left lower leg with palpable inguinal lymph nodes

Q/Q(M)-474423 Report a Problem

Which of the following melanoma scenarios have the best prognosis?

1 Twenty-one year old female with primary lesion located on the right lower leg

Increasing age and male gender have a negative effect on survival. Primary lesions located on the extremities have a better prognosis than those located on the trunk, head or neck region. Ulceration is also considered a poor prognostic factor. Furthermore, there is a significantly lower survival for those patients with palpable metastatic nodes (macrometastasis) when compared to those with micrometastatic nodes (nonpalpable).

Q/Q(M)-474423 Report a Problem

25

Page 26: Skin Tumor MCQ

Desmoplastic trichoepithelioma:

1 Is most common in middle-aged patients

2 More commonly appears in males than in females

3 Does not display foci of calcification or ossification

4 Presents as a well-circumscribed lesions located in the upper dermis

5 None of these answers are correct

Q/Q(M)-476104 Report a Problem

Desmoplastic trichoepithelioma:

4 Presents as a well-circumscribed lesions located in the upper dermis

Desmoplastic trichoepithelioma is a variant of trichoepithelioma, an uncommon adnexal tumor with differentiation toward hair structures. It is three times more common in females and usually occurs in young adults. The tumor is a well-circumscribed nodule lesions located in the upper dermis. Strands or columns of basaloid cells are seen surrounded by fibrotic or desmoplastic stroma. Horn cysts may also be seen, as well as foci of sebaceous cells, calcification and ossification.

Q/Q(M)-476104 Report a Problem

Which of the following immunologic drugs has been shown to increase survival of patients with stage III and stage IV melanoma?

1 Etanercept

2 Adalimumab

3 Ustekinumab

4 Ipilimumab

5 Infliximab

Q/Q(M)-482376 Report a Problem

Which of the following immunologic drugs has been shown to increase survival of patients with stage III and stage IV melanoma?

4 Ipilimumab

A recent study in the New England Journal of Medicine reported an increase in survival of patients with stage III

26

Page 27: Skin Tumor MCQ

and stage IV melanoma with a new immunologic therapy agent named ipilimumab. The survival was increased from 6 months to 10 months (P < 0.001). All the other medicines are not used to treat melanoma; they are used to treat psoriasis, among other disease.

Q/Q(M)-482376 Report a Problem

What would be the characteristic histopathologic findings of this lesion?

1 Full thickness keratinocytic atypia

2 Cornoid lamella

3 Pale staining cells

4 Horn pseudocysts

5 Wedge shaped granular layer with lichenoid infiltrate

Q/Q(M)-476650 Report a Problem

What would be the characteristic histopathologic findings of this lesion?

2 Cornoid lamella

Five clinical variants of porokeratosis are recognized. They are the classic porokeratosis of Mibelli, disseminated superficial actinic porokeratosis, prokeratosis palmaris et plantaris disseminata, linear porokeratosis, and punctate porokeratosis. On histopathology they have varying degrees of a cornoid lamella. This appears as a column of parakeratosis.

Q/Q(M)-476650 Report a Problem

Mohs micrographic surgery the treatment of choice for all of the following, except:

1 1 cm SCC located on the chest

27

Page 28: Skin Tumor MCQ

2 2 cms BCC on lower extremities

3 1 cm BCC on the eyelid

4 Morpheaform BCC on the cheek

5 Recurrent BCC on the chest

Q/Q(M)-474437 Report a Problem

Mohs micrographic surgery the treatment of choice for all of the following, except:

1 1 cm SCC located on the chest

SCCs that measure 1cm on the chest are not an indication for Mohs micrographic surgery. Lesions must measure 2cms on the chest and extremities, or ?1cm for those located on the face to be considered for Mohs surgery. Due to the aggressive growth patter and subclinical spread of morpheaform and recurrent BCCs are always and indication for Mohs surgery.

Q/Q(M)-474437 Report a Problem

Periungual Squamous cell carcinoma has been linked to which HPV type(s)?

1 6, 11

2 2, 4

3 16

4 13

5 8

Q/Q(M)-477402 Report a Problem

Periungual Squamous cell carcinoma has been linked to which HPV type(s)?

3 16

Infections associated with Human Papilloma Virus can produce growths on the epithelial or mucosal surfaces. There are over 100 strains of these viruses, and some of these strains can predispose to intraepithelial carcinomas, particularly when involving the anal or genital mucosa. In general, HPV strains 16 and 18 are classified as his risk virus types and can be associated with cervical cancer , oral cancer, anal cancer and periungual cancers. HPV 6 and 11 are associated with condyloma acuminata; HPV types 2, 4 are associated with common warts; HPV type 13 has been associated with Heck's disease and HPV type 8 has been associated with epidermal dysplasia verruciformis.

Q/Q(M)-477402 Report a Problem

What cancer in women most commonly metastasizes to the skin?

28

Page 29: Skin Tumor MCQ

1 Breast cancer

2 Medullary thyroid carcinoma

3 Glioblastoma multiforme

4 Colon adenocarcinoma

5 Cervical cancer

Q/Q(M)-476833 Report a Problem

What cancer in women most commonly metastasizes to the skin?

1 Breast cancer

Breast cancer is the most common cancer to metastasize to the skin in women.

Q/Q(M)-476833 Report a Problem

Dermatofibrosarcoma protuberans is:

1 Cytogenetically characterized by reciprocal translocation t(17;22)(q22;q13)

2 Factor XIIIa positive

3 CEA positive

4 CD 34 positive

5 Cytogenetically characterized by reciprocal translocation t(17;22)(q22;q13) and CD 34 positive

Q/Q(M)-474986 Report a Problem

Dermatofibrosarcoma protuberans is:

5 Cytogenetically characterized by reciprocal translocation t(17;22)(q22;q13) and CD 34 positive

A chromosomal reciprocal translocation t(17;22)(q22;q13), and supernumerary ring chromosome have been reported as cytogenetic characteristics of DFSP. Typically DFSP is CD34 positive and factor XIIIa negative, allowing its differentiation from dermatofibroma.

Q/Q(M)-474986 Report a Problem

What temperature must be achieved for adequate treatment of a small superficial squamous cell carcinoma with cryotherapy?

1 -10 degrees Celcius

29

Page 30: Skin Tumor MCQ

2 -20 degrees Celcius

3 -30 degrees Celcius

4 -40 degrees Celcius

5 -50 degrees Celcius

Q/Q(M)-479616 Report a Problem

What temperature must be achieved for adequate treatment of a small superficial squamous cell carcinoma with cryotherapy?

5 -50 degrees Celcius

Cryosurgery destroys tumor if it is frozen to -40 to -70 degrees Celcius, for at least 2 cycles with a 60 second thaw. Benign lesions can be destroyed at temperatures around -25 degrees Celcius.

Q/Q(M)-479616 Report a Problem

Which substance does p53 normally activate to promote apoptosis via inhibition of bcl-2?

1 p21

2 p16

3 Puma

4 Mdm2

5 Akt

Q/Q(M)-482363 Report a Problem

Which substance does p53 normally activate to promote apoptosis via inhibition of bcl-2?

3 Puma

p53 is the most commonly mutated tumor suppressor gene involved in human cancer and is often mutated in SCC and BCC. p53 acts via two main pathways, 1) activation of p21 (Cdk inhibitor) which leads to cell cycle arrest, and 2) activation of Puma which inhibits Bcl-2 (apoptosis inhibitor) thereby leading to cell death. CKDN2A, a gene that when mutated leads to a risk of melanoma, acts via 1) activation of p16 (another Cdk inhibitor) and 2) activation of p14ARF which inhibits Mdm2 (which normally degrades p53). Akt is involved in the PI3K-Akt signaling pathway and inhibits cell cycle arrest and apoptosis.

Q/Q(M)-482363 Report a Problem

All of the following are true regarding lentigo maligna except:

1 High rates of recurrence

30

Page 31: Skin Tumor MCQ

2 Occurs mostly on head and neck

3 Mostly in sun-exposed areas

4 Margins difficult to evaluate

5 Spares oral mucosa

Q/Q(M)-477197 Report a Problem

All of the following are true regarding lentigo maligna except:

5 Spares oral mucosa

Lentigo maligna is a type of melanoma in situ that usually presents as a poorly circumscribed, variably colored patch. It occurs on sun-exposed areas on the head and neck. Margins may be difficult to evaluate and recurrences are relatively common. There are rare reports of spread onto the oral mucosa as well as conjunctiva.

Q/Q(M)-477197 Report a Problem

A patient having Mohs surgery for a squamous cell carcinoma on the ear has tumor invading the cartilage. What stage disease does the patient have?

1 T0

2 T1

3 T2

4 T3

5 T4

Q/Q(M)-477190 Report a Problem

A patient having Mohs surgery for a squamous cell carcinoma on the ear has tumor invading the cartilage. What stage disease does the patient have?

5 T4

The staging of SCC�s are as follows. T0 lesions are in situ. T1 lesions are less than 2 cm in diameter. T2 lesions are 2-4 cm in diameter. T3 lesions are greater than 4 cm in diameter. T4 lesions are invasive of muscle, cartilage, or bone.

Q/Q(M)-477190 Report a Problem

What is the average time between a BCC primary tumor and its metastasis?

1 6 months

31

Page 32: Skin Tumor MCQ

2 1 year

3 2-4 years

4 9 years

5 greater than 15 years

Q/Q(M)-482917 Report a Problem

What is the average time between a BCC primary tumor and its metastasis?

4 9 years

The size of a BCC is related to its metastasis risk. Invasion of bone, cartilage, and muscle is not common, however, spread does occur along perichondrium, periosteum, fascia, or tarsal plate.

Q/Q(M)-482917 Report a Problem

What is the most common location for this rapidly growing tumor?

1 Head and neck

2 Chest

3 Back

4 Arms

5 Legs

Q/Q(M)-476841 Report a Problem

What is the most common location for this rapidly growing tumor?

1 Head and neck

Merkel cell carcinoma a cutaneous neuroendocrine carcinoma that usually occurs on the head and neck. It is a high grade malignant tumor with a 5-year mortality rate of 30-64%. On histopathology, there are trabecular strands of

32

Page 33: Skin Tumor MCQ

basophilic cells that stain with a characteristic paranuclear dot pattern with cytokeratin 20.

Q/Q(M)-476841 Report a Problem

An elderly man presents to your office with a telangiectatic, violaceous 1cm dome-shaped nodule on the neck. Biopsy reveals large, solid nests of cells of intermediate size, with a trabecular pattern at the periphery. These cells involve the dermis and spread into the subcutaneous fat, but spare the overlying epidermis. The cells are round and uniform in size, with a round to oval nucleus, small nucleoli, and evenly dispersed chromatin. Numerous mitotic figures and necrotic areas are seen. Neuron specific enolase is positive. Which of the following is true regarding this patient's diagnosis?

1 Mohs micrographic surgery is contraindicated in treatment of this lesion

2 Vimentin and desmin stains may be positive

3 S-100 stains should be positive

4 This lesion may contain ACTH

5 The neoplasm should not contain gastrin

Q/Q(M)-476106 Report a Problem

An elderly man presents to your office with a telangiectatic, violaceous 1cm dome-shaped nodule on the neck. Biopsy reveals large, solid nests of cells of intermediate size, with a trabecular pattern at the periphery. These cells involve the dermis and spread into the subcutaneous fat, but spare the overlying epidermis. The cells are round and uniform in size, with a round to oval nucleus, small nucleoli, and evenly dispersed chromatin. Numerous mitotic figures and necrotic areas are seen. Neuron specific enolase is positive. Which of the following is true regarding this patient's diagnosis?

4 This lesion may contain ACTH

This patient has a Merkel cell carcinoma. Vimentin, desmin, and S-100 are consistently absent in MCC. Mohs micrographic surgery has been used successfully for the treatment of MCC, with or without adjuvant therapy. This neoplasm may sometimes contain several neuropeptides including vasoactive intestinal peptide, calcitonin, ACTH, gastrin, and somatostatin.

Q/Q(M)-476106 Report a Problem

What is the most common location for a fibroepithelioma of pinkus variant of BCC is

1 head and neck

2 extremities

3 hands and feet

4 upper trunk

33

Page 34: Skin Tumor MCQ

5 lumbosacral region

Q/Q(M)-482908 Report a Problem

What is the most common location for a fibroepithelioma of pinkus variant of BCC is

5 lumbosacral region

Fibroepithelioma of pinkus variant of BCC is often a fleshy or pink colored nodule. The most common location of a fibroepithelioma of pinus is the lumbosacral region. Superficial variants are more common on the trunk and extremities. BCC makes up the most common non melanotic skin cancer at 75%.

Q/Q(M)-482908 Report a Problem

All of the following statements regarding the patient pictured are true EXCEPT:

1 Spina bifida may be an associated finding

2 The patient likely has a mutation in the PTCH gene

3 Ameloblastoma is a tumor associated with this disease

4 This patient likely had a similarly affected parent

5 These lesions appeared in childhood

Q/Q(M)-474911 Report a Problem

All of the following statements regarding the patient pictured are true EXCEPT:

3 Ameloblastoma is a tumor associated with this disease

The patient has basal cell nevus syndrome, or Gorlin syndrome, which is characterized by the appearance of multiple basal cell carcinomas in childhood, odontogenic keratocysts of the jaw, and skeletal defects (i.e macrocephaly, hypertelorism, frontoparietal bossing, spina bifida, or rib abnormalities). Tumors associated with this disease include medulloblastoma and meningioma (not ameloblastoma). It is caused by a mutation in the PTCH gene located on 9q22 and is inherited in an autosomal dominant manner.

Q/Q(M)-474911 Report a Problem

The most common site for intra oral melanoma is?

34

Page 35: Skin Tumor MCQ

1 buccal

2 soft palate

3 hard palate

4 gingiva

5 uvula

Q/Q(M)-482205 Report a Problem

The most common site for intra oral melanoma is?

3 hard palate

Multiple case series have shown that the hard palate (and specifically the anterior hard palate/alveolar arch) is the highest risk location for intra-oral melanoma. Prognosis is generally worse for oral melanoma, which is usually due to delay in diagnosis and presentation with more invasive disease.

Q/Q(M)-482205 Report a Problem

Which of the following are features of dermatofibromas, which help differentiate it from dermatofibrosarcoma protuberans?

1 Commonly located on the extremities

2 Factor XIIIa negative

3 Factor XIIIa positive

4 Commonly located on the trunk

5 Commonly located on the extremities and factor XIIIa positive

Q/Q(M)-474989 Report a Problem

Which of the following are features of dermatofibromas, which help differentiate it from dermatofibrosarcoma protuberans?

5 Commonly located on the extremities and factor XIIIa positive

Typically DFSP is CD34 positive and factor XIIIa negative, allowing its differentiation from dermatofibroma.

Q/Q(M)-474989 Report a Problem

The most common location for primary mucinous carcinoma is:

1 neck

35

Page 36: Skin Tumor MCQ

2 eyelid

3 areola

4 scrotum

5 nose

Q/Q(M)-482483 Report a Problem

The most common location for primary mucinous carcinoma is:

2 Eyelid

Mucinous carcinoma presents as a slowly growing, asymptomatic, round, erythematous nodule on the head and neck. Forty percent of cases occur on the eyelid. Histologically it is characterized by the presence of large areas of mucin (âsea o� f mucous) with small islands of basophilic epithelial cells. Primar�� y mucinous carcinoma of the skin has an indolent course. Local recurrence occurs in 1/3 of patients following excision. The rate of metastasis is low (9.6%).

Q/Q(M)-482483 Report a Problem

What is the most common location of oral SCC?

1 Soft palate

2 Buccal mucosa

3 Gingiva

4 Dorsal tongue

5 Lateral tongue

Q/Q(M)-476592 Report a Problem

What is the most common location of oral SCC?

5 Lateral tongue

Squamous cell carcinomas are the most common carcinoma of the oral cavity. The most common locations for this tumor are the lateral and ventral surfaces of the tongue and the floor of the mouth.

Q/Q(M)-476592 Report a Problem

Dermoscopic features suggestive of malignant melanoma include:

1 Presence of 2 or more colors within the lesion

2 Blue-whitish veil

36

Page 37: Skin Tumor MCQ

3 Asymmetric radial streaming

4 Abrupt interruption of pigment network in the periphery

5 All of these answers are correct

Q/Q(M)-474426 Report a Problem

Dermoscopic features suggestive of malignant melanoma include:

5 All of these answers are correct

Asymmetry, multicomponent pattern, blue-whitish veil, parallel-ridge pattern, atypical pigment network, uneven radial streaming, localized irregular and diffuse pigmentation, irregularly distributed globules, and regression structures are all dermoscopic features suggestive of malignant melanoma.

Q/Q(M)-474426 Report a Problem

All of the following are risk factors for metastasis from a primary squamous cell carcinoma EXCEPT:

1 Increasing tumor size

2 Location on the ear

3 Recurrent tumor

4 Tumor within ulcer

5 All of these answers are correct

Q/Q(M)-476123 Report a Problem

All of the following are risk factors for metastasis from a primary squamous cell carcinoma EXCEPT:

5 All of these answers are correct

The risk of metastasis from a primary SCC increases with tumor size (>2cm), location (lips, ear, and eyelid, among others), depth of invasion, immunosuppression, degree of differentiation, and perineural invasion. Recurrent tumors and SCCs arising in areas of chronic inflammation (such as osteomyelitis, burn scars, or ulcers) are also considered high risk for metastases.

Q/Q(M)-476123 Report a Problem

Mutations of the p53 gene has been associated with the development of:

1 Mmelanoma

2 Kaposi�s sarcoma

3 Actinic keratosis

37

Page 38: Skin Tumor MCQ

4 Molluscum contagiosum

5 All of these answers are correct

Q/Q(M)-474448 Report a Problem

Mutations of the p53 gene has been associated with the development of:

3 Actinic keratosis

UVB radiation triggers the formation of thymidine dimers both in DNA and RNA, resulting in mutated keratinocytes. The mutations occur on the tumor suppressor gene p53 within the keratinocytes resulting in impairment of the mechanism of apoptosis. Therefore, clonal expansion of mutated keratinocytes may occur leading to the formation of AKs.

Q/Q(M)-474448 Report a Problem

Which is the most common neoplasm in patients who have had long-term PUVA therapy?

1 Basal cell carcinoma

2 Atypical fibroxanthoma

3 Squamous cell carcinoma

4 Cutaneous T cell lymphoma

5 Melanoma

Q/Q(M)-477325 Report a Problem

Which is the most common neoplasm in patients who have had long-term PUVA therapy?

3 Squamous cell carcinoma

The most common neoplasm to arise in patients who have been treated with long-term PUVA therapy is squamous cell carcinomas. In a study by Stern, et.al. a 12.8 fold risk was found for the development of squamous cell cancers in patients who received high doses than those that received low doses.

Q/Q(M)-477325 Report a Problem

What is the most common location for an epitheloid sarcoma?

1 head and neck

2 proximal extremities

3 hands and forearms

4 lower legs

38

Page 39: Skin Tumor MCQ

5 groin and buttocks

Q/Q(M)-482523 Report a Problem

What is the most common location for an epitheloid sarcoma?

3 hands and forearms

Epithelioid sarcoma is a rare soft tissue sarcoma that most commonly develops in young adults, males greater than females, with a predilection for the distal upper extremities, namely hands and forearms. Most tumors present as firm-to-hard palpable masses, either in the deep soft tissue or in the dermis. The superficial lesions can present with ulceration. Five year survival and ten year survival rate for patients with epithelioid sarcoma are approximately 50-70% and 42-55% respectively (Journal of bone and Joint Surgery (Am), 70-A: 862-870, 1988). Female patients have a more favorable outcome. Proximal lesions have been shown to have worse outcomes compared to distal lesions.

Q/Q(M)-482523 Report a Problem

Chloroma is a characteristic cutaneous manifestation of:

1 Tuberous sclerosis

2 Sweet�s syndrome

3 Neurofibromatosis

4 Leukemia

5 Pseudomonas sepsis

Q/Q(M)-477324 Report a Problem

Chloroma is a characteristic cutaneous manifestation of:

4 Leukemia

Chloromas, also termed granulocytic sarcomas, are a localized tumor composed of immature granulocytic cells. They frequently have a greenish coloration due to the presence of myeloperoxidase and most commonly affect the bone. The condition most often occurs in patients with acute leukemia of the myeloid type.

Q/Q(M)-477324 Report a Problem

Which of the following is the most important prognostic indicator in a patient with cutaneous lymphoma?

1 Age

2 Primary vs secondary cutaneous

3 Gender

39

Page 40: Skin Tumor MCQ

4 Extent of cutaneous involvement

5 Subtype of lymphoma

Q/Q(M)-477176 Report a Problem

Which of the following is the most important prognostic indicator in a patient with cutaneous lymphoma?

2 Primary vs secondary cutaneous

When staging lymphoma, it is critical to determine whether the lymphoma is primary cutaneous arising in the skin or secondary cutaneous arising in association with nodal or extranodal tumor. The prognosis is worse in secondary when compared to primary lymphomas, irrespective of histologic diagnosis.

Q/Q(M)-477176 Report a Problem

Denileukin diftitox (ONTAK�):

1 is a systemic treatment option for CTCL

2 is a diphtheria fusion toxin

3 targets the interleukin-2 receptor

4 None of these answers are correct

5 All of these answers are correct

Q/Q(M)-474430 Report a Problem

Denileukin diftitox (ONTAK�):

5 All of these answers are correct

Denileukin diftitox is as diphtheria fusion toxin that targets the IL-2 receptor. It is a systemic treatment alternative for recalcitrant or advance CTCL.

Q/Q(M)-474430 Report a Problem

All of the following are true regarding curettage of giant congenital melanocytic nevi except:

1 Treatment remains a controversial topic

2 Best performed during the first 2 weeks of life

3 Decreases the risk of melanoma

4 Offers an adequate alternative to surgical excision

40

Page 41: Skin Tumor MCQ

5 Mandates careful long-term follow-up

Q/Q(M)-477360 Report a Problem

All of the following are true regarding curettage of giant congenital melanocytic nevi except:

3 Decreases the risk of melanoma

Curettage of giant congenital nevis is a highly controversial treatment option. De Raeve and Roseeuw reported on 16 neonates who underwent curettage for treatment of their giant congenital nevi. They noted that it was best performed within the first two weeks of life. They claim that it offers an adequate cosmetic alternative to surgical excision. The risk of melanoma developing within the giant congenital nevus is not decreased and long-term follow-up is essential.

Q/Q(M)-477360 Report a Problem

The Gorlin syndrome is characterized by:

1 Multiple BCCs during childhood

2 Macrocephaly

3 Odontogenic keratocysts of the jaw

4 Autosomal-recessive inheritance pattern

5 All of these answers are correct except Autosomal-recessive inheritance pattern

Q/Q(M)-474416 Report a Problem

The Gorlin syndrome is characterized by:

5 All of these answers are correct except Autosomal-recessive inheritance pattern

Gorlin�s syndrome is characterized by the appearance of multiple BCCs during childhood, odontogenic keratocysts of the jaw, and skeletal defects. It is inherited in an autosomal dominant pattern.

Q/Q(M)-474416 Report a Problem

A male teenager had this skin lesion on the oral exam. He did not have any symptoms associated with it. He was found to have similar lesions on upper extremities. What is the syndrome most likely associated with this finding?

1 Turner syndrome

2 Noonan syndrome

3 Beckwith-Wiedemann syndrome

4 Primary amyloidosis

41

Page 42: Skin Tumor MCQ

5 MEN type Iia

Q/Q(M)-482877 Report a Problem

A male teenager had this skin lesion on the oral exam. He did not have any symptoms associated with it. He was found to have similar lesions on upper extremities. What is the syndrome most likely associated with this finding?

2 Noonan syndrome

This is a granular cell tumor. Patient had several lesions on upper extremities. Biopsy confirmed he had multiple granular cell tumors. Multiple granular cell tumors in kids are associated with Noonan syndrome and Neurofibromatosis. They can occur anywhere, but more frequently on the tongue, upper extremities and trunk.

Q/Q(M)-482877 Report a Problem

What is the mean time-frame for development of Stewart-Treves Syndrome?

1 Less than 1 year

2 1 year

3 5 years

4 10 years

5 >20 years

Q/Q(M)-482497 Report a Problem

What is the mean time-frame for development of Stewart-Treves Syndrome?

5 >20 years

Stewart-Treves syndrome refers to an angiosarcoma arising from chronic lymphedema. It typically occurs as a complication of long-lasting lymphedema of the arm after mastectomy and/or radiotherapy for breast cancer. It is a very late complication of disease, usually developing several years later. The mean time-frame for development of Stewart-Treves Syndrome in the literature is 23 years (4-40 years).

Q/Q(M)-482497 Report a Problem

Knowing that the patient in this picture has a biopsy confirmed BCC, which of the following would be the treatment of choice:

1 Conventional excision

2 Imiquimod

3 Radiation

4 Mohs micrographic surgery

42

Page 43: Skin Tumor MCQ

5 Cryosurgery

Q/Q(M)-474905 Report a Problem

Knowing that the patient in this picture has a biopsy confirmed BCC, which of the following would be the treatment of choice:

4 Mohs micrographic surgery

Providing the highest cure rates, Mohs micrographic surgery is the treatment of choice for large BCCs (?1cm on the face or ?2 cms on the trunk), located on high-risk anatomic areas (ear, eyelids, lips, genitals, nose, temples). , Morpheaform BCCs, tumors with positive margins after conventional excision, and recurrent BCCs, are also an indication for Mohs surgery.

Q/Q(M)-474905 Report a Problem

Which of the following is commony seen in seborrheic keratoses when examined with dermoscopy?

1 Maroon lagoons

2 Network

3 Pigment globules

4 Milia like cysts

5 Blue-gray veil

Q/Q(M)-474393 Report a Problem

Which of the following is commony seen in seborrheic keratoses when examined with dermoscopy?

4 Milia like cysts

The main dermoscopic features of seborrheic keratoses include comedolike openings, fissures, milialike cysts, fingerprinting and lack of true pigment network. In fact, it is the lack of true pigment network, branched streaks, and pigment globules that differentiate SKs from melanocytic lesions.

Q/Q(M)-474393 Report a Problem

43

Page 44: Skin Tumor MCQ

What is the eponym for this metastasis to the umbilicus?

1 Tripe palm

2 Sign of Leser-Trelat

3 Pityriasis rotunda

4 Trousseau syndrome

5 Sister Mary Joseph Nodule

Q/Q(M)-476847 Report a Problem

What is the eponym for this metastasis to the umbilicus?

5 Sister Mary Joseph Nodule

Sister Mary Joseph nodule is a cutaneous metastasis that is most commonly associated with stomach, large bowel, ovary, and pancreatic cancer. Tripe palms are most commonly associated with lung, Leser-Trelat with adenocarcinomas of the stomach, colon and breast, Trousseau Sign with carcinoma of the pancreas and pityriasis rotunda with hepatocellular carcinoma and gastric cancer.

Q/Q(M)-476847 Report a Problem

A patient has a malignant melanoma 1.6mm thick with ulceration and a micrometastasis in 1 node. The patient's staging according to the American Joint Committee on Cancer Staging System is:

1 IIC

2 IIIA

3 IIIB

4 IIIC

5 IV

Q/Q(M)-476101 Report a Problem

44

Page 45: Skin Tumor MCQ

A patient has a malignant melanoma 1.6mm thick with ulceration and a micrometastasis in 1 node. The patient's staging according to the American Joint Committee on Cancer Staging System is:

3 IIIB

An ulcerated tumor of any size with micrometastasis in 1 node is T1-4b N1a M0. This corresponds to stage IIIB.

Q/Q(M)-476101 Report a Problem

Desmoplastic trichoepitheliomas are commonly located on:

1 Legs

2 Chest

3 Back

4 Face

5 Scalp

Q/Q(M)-474401 Report a Problem

Desmoplastic trichoepitheliomas are commonly located on:

4 Face

Desmoplastic trichoepithelioma presents as small, firm, umbilicated papule on the face of young adults.

Q/Q(M)-474401 Report a Problem

Sixty-year old, male patient that presents with a �stuck on�, waxy, hyperkeratotic and hyperpigmented plaque on his back. The most likely diagnosis is:

1 Actinic keratosis

2 SCC in situ

3 Angiosarcoma

4 Seborrheic keratosis

5 None of these answers are correct

Q/Q(M)-474900 Report a Problem

Sixty-year old, male patient that presents with a �stuck on�, waxy, hyperkeratotic and hyperpigmented plaque on his back. The most likely diagnosis is:

4 Seborrheic keratosis

45

Page 46: Skin Tumor MCQ

The diagnosis is SK. Notice the warty surface of the lesion with a �stuck on� appearance.

Q/Q(M)-474900 Report a Problem

A patient with multiple skin nodules has a biopsy suggestive of cutaneous lymphoma. The next appropriate step in making the diagnosis is:

1 Polymerase chain reaction

2 Complete blood count

3 Flow cytometry

4 Immunohistochemical stains

5 Chest xray

Q/Q(M)-477146 Report a Problem

A patient with multiple skin nodules has a biopsy suggestive of cutaneous lymphoma. The next appropriate step in making the diagnosis is:

1 Polymerase chain reaction

In a patient with skin lesions and pathology suggestive of cutaneous lymphoma, gene rearrangement studies should be done to detect a clonal population of lymphocytes. Polymerase chain reaction (PCR) is approximately 1000 times more sensitive than Southern blot in the detection of a clonal population of cells. In addition, PCR may be used on formalin fixed tissue as well as fresh tissue, whereas Southern blot testing must be done of fresh tissue only.

Q/Q(M)-477146 Report a Problem

What would you expect to see under dermoscopy of this vascular neoplasm?

1 Red sacculae

2 Arborizing blood vessels

3 Hair pin telangectasia

4 Milky red globules

5 Blue-grey ovoid nests

Q/Q(M)-476849 Report a Problem

46

Page 47: Skin Tumor MCQ

What would you expect to see under dermoscopy of this vascular neoplasm?

1 Red sacculae

Hemangioma have a characteristic appearance under dermoscopy. Typically, they have a maroon lagoon or red sacculae appearance.

Q/Q(M)-476849 Report a Problem

The human papilloma virus type associated with red brown smooth and warty papules is

1 HPV 1

2 HPV 5

3 HPV 7

4 HPV 13

5 HPV 16

Q/Q(M)-476654 Report a Problem

The human papilloma virus type associated with red brown smooth and warty papules is

5 HPV 16

Bowenoid papulosis manifests clinically as multiple red-brown warty papules or confluent planques on the external genitalia. These lesions may resemble genital warts but histologically represent high-grade squamous intraepithelial lesions. Bowenoid papulosis is caused by infection by HPV and linked to HPV 16, 18, 31, 35, and 39.

Q/Q(M)-476654 Report a Problem

40 year-old male presented with a slow growing large tumor on the back. The patient undergoes wide excision with adjunctive chemotherapy. Which of the chemotherapeutic agents was used?

47

Page 48: Skin Tumor MCQ

1 Imatinib

2 Cytarabine

3 Doxorubicin

4 Vincristine

5 Cisplatin

Q/Q(M)-478125 Report a Problem

40 year-old male presented with a slow growing large tumor on the back. The patient undergoes wide excision with adjunctive chemotherapy. Which of the chemotherapeutic agents was used?

1 Imatinib

Imatinib or Gleevec is used in the treatment of primary or locally recurrent dermatofibrosarcoma protuberans. Even with wide excision, the 5 year recurrence rate for this tumor can be 20-25%. Activation of the Platelet derived growth factor receptor associated with overexpression of Platelet Derived growth factor is central to the development of DFSP Imatinib is a protein tyosine kinase inhibitor for Bcr-Abl kinase and is used primary in chronic myelogenous leukemia with the Philadelphia chromosome defect. Imatinib also inhibits the typosine kinases associated with platelet derived growth factor and stem cell factor. Therefore, it directly inhibits the Platelet Derived Growth factor receptor signaling cascade,that plays a critical role in the pathogenesis and growth of DFSPs. While Imatinib has been successful in clinical trials, it is not yet FDA approved for the treatment of DFSPs. The other listed agents are not adjunctive or primary therapies for DFSP.

Q/Q(M)-478125 Report a Problem

Characteristics indicative of a high risk of metastasis with SCC include:

1 Mitotic rate

2 Extremity location

3 Well differentiated

4 Deep invasion

5 Papillomavirus infection

Q/Q(M)-478728 Report a Problem

Characteristics indicative of a high risk of metastasis with SCC include:

4 Deep invasion

The rate of SCC metastasis from all skin sites ranges from 0.5% to 5.2%. Local recurrences and metastasis are related to: 1.treatment modality, 2. prior hitological, 3. location, 4. size, 5.depth, 6. histological differentiation, 7. histological evidence of perineural involvement, 8. histological evidence of desmoplastic features, 9. precipitating

48

Page 49: Skin Tumor MCQ

factors other than UV light, and 10. host immunosuppression. In reference to metastatic disease, the highest rates occur from scars, the lip, and the external ear. Patients with perineural spread have a local recurrence rate of 47.2% and a metastatic rate of 34.8%. Demosplastic SCC's are 6 times more likely to metastasize than other histological patterns.

Q/Q(M)-478728 Report a Problem

The patient is a 45 year old male complaining of red, chapped lower lip. Which of the following lasers is the most appropriate to treat this condition?

1 Pulsed Dye Laser

2 Nd:YAG laser

3 CO2 laser

4 Diode laser

5 Laser treatment is not an option

Q/Q(M)-474901 Report a Problem

The patient is a 45 year old male complaining of red, chapped lower lip. Which of the following lasers is the most appropriate to treat this condition?

3 CO2 laser

The patient has actinic cheilitis. Notice the red, scaly lower lip, with erosions and fissures. The CO2 laser is currently a common treatment alternative for this condition.

Q/Q(M)-474901 Report a Problem

A patient with a innumerable disseminated keratoacanthomas, including lesions on the larynx and oral mucosa:

1 Is unlikely to have palmoplantar involvement

2 Likely has an underlying immune deficiency

3 Is at high risk for myelodysplasia

49

Page 50: Skin Tumor MCQ

4 Likely inherited their condition in an autosomal dominant manner

5 Likely developed them during adulthood

Q/Q(M)-476098 Report a Problem

A patient with a innumerable disseminated keratoacanthomas, including lesions on the larynx and oral mucosa:

5 Likely developed them during adulthood

This patient has the Grzybowski type of keratoacanthomas. Typically diagnosed in adulthood, these patients have the sudden appearance of hundreds of small lesions in a disseminated fashion. The lesions can be found anywhere on the body including palms, soles, larynx, and oral mucosa.

Q/Q(M)-476098 Report a Problem

All of the following statements regarding Bowen's disease are true EXCEPT:

1 Lesions arising on the lower limbs are more common in men than in women

2 The basement membrane remains intact on histopathology

3 5% of patients with Bowen's disease develop invasive squamous cell carcinoma

4 Chronic sun exposure is a risk factor for Bowen's disease

5 Mucosal Bowen's disease may appear as a verrucous plaque

Q/Q(M)-476125 Report a Problem

All of the following statements regarding Bowen's disease are true EXCEPT:

1 Lesions arising on the lower limbs are more common in men than in women

The most common locations for BD include the head and neck regions and the extremities. BD arising on the lower limbs is frequently found in women, whereas lesions located on the ears and scalp are more common in men. The basement membrane remains intact on histopathology. 5% of patients develop invasive SCC. Risk factors include chronic sun exposure, immunosuppression, HPV, arsenic exposure, and ionizing radiation. Mucosal BD may appear as a verrucous or polypoid plaque, as an erythroplakic patch, or as a velvety red plaque.

Q/Q(M)-476125 Report a Problem

A 60 y/o male smoker with multiple sclerosis presents for evaluation of painful 5cm x 4cm plaque on right posterior shoulder, which appeared over 3 months. What is the most likely diagnosis?

1 Irritated Seborrheic Keratosis

2 Squamous Cell Carcinoma

3 Basal Cell Carcinoma

50

Page 51: Skin Tumor MCQ

4 Metastatic Carcinoma

5 Malignant Melanoma

Q/Q(M)-482886 Report a Problem

A 60 y/o male smoker with multiple sclerosis presents for evaluation of painful 5cm x 4cm plaque on right posterior shoulder, which appeared over 3 months. What is the most likely diagnosis?

3 Basal Cell Carcinoma

Basal cell carcinoma. It has a male to female ratio 2:1. UVB damages DNA and produces a C-T transition mutation (65% of BCC). P53 and PTCH genes are commonly affected. It has a slow, indolent growth potential: metastatic potential 0.0028% to 0.1%.; known to metastasize to lymph nodes and lungs.

Q/Q(M)-482886 Report a Problem

Which of the following markers do not stain melanocytic lesions:

1 Neuron specific enolase

2 Vimentin

3 S-100

4 HMB-45

5 All of these answers are correct

Q/Q(M)-474449 Report a Problem

Which of the following markers do not stain melanocytic lesions:

1 Neuron specific enolase

Neuron specific enolase is positive in Merkel cell carcinoma. Vimentin, S-100 and HMB-45 stains melanocytic lesions.

Q/Q(M)-474449 Report a Problem

Which of the following best describes the incidence of skin cancer in transplant recipients from greatest to least?

1 BCC>melanoma>SCC>Merkel cell

2 SCC>BCC>melanoma>Merkel cell

3 BCC>SCC>melanoma>Merkel cell

4 SCC>BCC>Merkel cell>melanoma

51

Page 52: Skin Tumor MCQ

5 Merkel cell>SCC>melanoma>BCC

Q/Q(M)-478725 Report a Problem

Which of the following best describes the incidence of skin cancer in transplant recipients from greatest to least?

2 SCC>BCC>melanoma>Merkel cell

The incidence of skin cancer in transplant recipient (from greatest to least) is as follows: SCC>BCC>melanoma>Merkel cell.

Q/Q(M)-478725 Report a Problem

Histologically, this lesion is shows plump, polygonal cells arranged in nests and fascicles with granular cytoplasm. Which immunohistochemical stain would be positive?

1 Colloidal iron

2 Von kossa

3 Warthin-starry

4 S-100

5 CD68

Q/Q(M)-476839 Report a Problem

Histologically, this lesion is shows plump, polygonal cells arranged in nests and fascicles with granular cytoplasm. Which immunohistochemical stain would be positive?

4 S-100

Granular cell tumors are benign growths which typically occur on the tongue. They are typically well-circumscribed, raised, firm nodules. Histologically, the cells have uniform nuclear characteristics and granular cytoplasm due to presence of lysozyme. The lesions are PAS positive and S-100 positive.

Q/Q(M)-476839 Report a Problem

52

Page 53: Skin Tumor MCQ

What is the diagnosis?

1 Condyloma acuminata

2 Molloscum contagiosum

3 Pearly penile papules

4 Lichen planus

5 Psoriasis

Q/Q(M)-476845 Report a Problem

What is the diagnosis?

3 Pearly penile papules

Pearly penile papules are benign, filiform papules that form along the corona of the penis. THey are often mistaken for a form of veneral disease. Histologically, they are similar to angiofibromas and show ectatic vessels surrounded by dense connective tissue.

Q/Q(M)-476845 Report a Problem

A 55 year-old female presents with an ulcerated malignant melanoma with Breslow dept of 1.5mm. Sentinel lymph node biopsy is negative. Which of the following is the correct staging classification?

1 Stage IB

2 Stage IIA

3 Stage IIB

4 Stage IIIA

5 Stage IIIB

Q/Q(M)-477399 Report a Problem

A 55 year-old female presents with an ulcerated malignant melanoma with Breslow dept of 1.5mm. Sentinel lymph node biopsy is negative. Which of the following is the correct staging classification?

53

Page 54: Skin Tumor MCQ

2 Stage IIA

The current melanoma staging is developed by the American Joint Comittee on Cancer. This system is based on four characterisitics: 1. Tumor thickness 2.Presence of ulceration within the primary tumor, 3. Involvement of lymph nodes and 4.presence of distal metastases. Stage 1A characterizes a melanoma that has less than 1mm in tumor thickness and no ulceration, no nodes, and no distant mets. Stage IB characterizes a melanoma with tumor thickness less than 1mm however with ulceration and no nodes or distal metastases. Stage IB also can characterize melanomas with tumor thickness between 1.01-2.00 mm without ulceration. Stage IIA classifies tumor with thickness between 1.01-2.00mm, with ulceration, and no nodes and no distal metastases. Stage IIA also characterizes those melanomas with thickness between 2.01-4.0mm without ulceration, nodes, or distal mets. Stage III and IV melanomas have nodal and distal met involvement respectively.

Q/Q(M)-477399 Report a Problem

Which neoplasm is associated with the Stewart-Treves syndrome?

1 Renal leiomyomas

2 Basal cell carcinoma

3 Angiosarcoma

4 Keratoacanthoma

5 T cell lymphoma

Q/Q(M)-477129 Report a Problem

Which neoplasm is associated with the Stewart-Treves syndrome?

3 Angiosarcoma

Stewart-Treves syndrome is the development of angiosarcoma in the setting of chronic lymphedema. Originally, named after radical mastectomy for the treatment of breast cancer. The term applies to the development of angiosarcoma in any chronic lymphedematous condition.

Q/Q(M)-477129 Report a Problem

Mutations in which gene would likely be found in the neoplastic cells of this lesion?

1 PATCH

2 p53

3 Fumarate hydratase

4 CREBBP

5 p63

54

Page 55: Skin Tumor MCQ

Q/Q(M)-476653 Report a Problem

Mutations in which gene would likely be found in the neoplastic cells of this lesion?

2 p53

Squamous cell carcinoma is the second most common cancer of the skin. Mutations in the tumor suppressor p16 and p53 are commonly found in SCC's. Normally, UV damage upregulates p53 thereby delaying cell cycle progression. DNA damage can then be repaired or the cell could undergo apoptosis. In squamous cell carcinoma, p53 exhibits loss of heterozygosity due to C to T or CC to TT mutations.

Q/Q(M)-476653 Report a Problem

All of the following may increase the incidence of SCC, EXCEPT:

1 Sun exposure

2 Immunosuppression

3 Increasing age

4 Proximity to the equator

5 All of the following may increase the incidence of SCC

Q/Q(M)-474434 Report a Problem

All of the following may increase the incidence of SCC, EXCEPT:

5 All of the following may increase the incidence of SCC

The incidence of SCC increases with age, and it is 35 times higher in individuals older than 75 years of age when compared to ages 50-55. The incidence doubles for each 8 to 10 degree decline in latitude, therefore, individuals living closer to the equator have a greater risk.

Q/Q(M)-474434 Report a Problem

The incidence of cutaneous squamous cell carcinoma in organ transplant recipients is increased by how much compared with the general population?

55

Page 56: Skin Tumor MCQ

1 2 fold

2 5 fold

3 10 fold

4 20 fold

5 65 fold

Q/Q(M)-477368 Report a Problem

The incidence of cutaneous squamous cell carcinoma in organ transplant recipients is increased by how much compared with the general population?

5 65 fold

The risk of skin cancer in organ transplant patients is dramatically increase and may be more aggressive. In one study, SCC�s had a 65x increased incidence, BCC 10x, and melanoma 3.4x.

Q/Q(M)-477368 Report a Problem

Which of the following statements regarding prognosis for malignant melanoma is true?

1 Increasing age has a positive effect on survival.

2Patients with primary lesions located on the extremities (except acral lesions) have a worse prognosis than those with tumors located on the trunk.

3There is a higher survival rate for patients with palpable metastatic nodes compared to those with micrometastatic nodal disease.

4For stage IV disease, patients with non-visceral metastases (eg skin, subcutis, distant lymph nodes) have a better prognosis compared with those with visceral metastases.

5 Male gender has a positive effect on survival.

Q/Q(M)-476727 Report a Problem

Which of the following statements regarding prognosis for malignant melanoma is true?

4For stage IV disease, patients with non-visceral metastases (eg skin, subcutis, distant lymph nodes) have a better prognosis compared with those with visceral metastases.

Increasing age and male gender have a negative effect on survival in melanoma. Patients with primary lesions located on the extremities (except acral lesions) have a better prognosis than those with tumors located on the head, neck, or trunk. There is a significantly lower survival for patients with palpable metastatic nodes (macrometastasis) when compared to those with micrometastatic nodes. For stage IV disease, patients with non-visceral metastases (eg skin, subcutis, distant lymph nodes) have a better prognosis compared with those with visceral metastases.

56

Page 57: Skin Tumor MCQ

Q/Q(M)-476727 Report a Problem

Which of the following is a clinical stage of keratoacanthomas?

1 Plaque

2 Mature

3 Proliferative

4 Resolving

5 All of these answers are correct except plaque

Q/Q(M)-474445 Report a Problem

Which of the following is a clinical stage of keratoacanthomas?

5 All of these answers are correct except plaque

Keratoacanthomas have three, consecutive, clinical stages: proliferative, mature and resolving. The proliferative stage is characterized by the appearance of a rapid growing papule. This phase is followed by the mature stage when the lesion acquires its characterisitic dome-shaped appearance with a central, keratinous core. Tumor resorption occurs during the involution stage resulting in a slightly depressed, hypopigmented scar.

Q/Q(M)-474445 Report a Problem

Which of the following melanoma subtypes is more consistently seen in dark-skinned individuals?

1 Acral lentiginous melanoma

2 Superficial spreading melanoma

3 Lentigo maligna melanoma

4 Nodular melanoma

5 Superficial spreading melanoma and Lentigo maligna melanoma

Q/Q(M)-474428 Report a Problem

Which of the following melanoma subtypes is more consistently seen in dark-skinned individuals?

1 Acral lentiginous melanoma

Acral lentiginous melanoma is the predominant type of melanoma in dark-skinned individuals. It is usually located on the soles, palms, and subungeal region of patients in their fifth to sixth decade of life.

Q/Q(M)-474428 Report a Problem

57

Page 58: Skin Tumor MCQ

What is the most location for this lesion which lacks phosphorylase in the epidermal cells?

1 Back

2 Buttock

3 Leg

4 Neck

5 Sacrum

Q/Q(M)-476836 Report a Problem

What is the most location for this lesion which lacks phosphorylase in the epidermal cells?

3 Leg

Clear cell acanthomas are slow growing, benign growths which typically occur on the leg. They have a striking histologic appearance in the epidermis with sharp demaracation and enlarged, pale cells. Excess glycogen in the cells accounts for their clear appearance and is due to a defect in phosphorylase.

Q/Q(M)-476836 Report a Problem

A child presents with a giant congenital nevus overlying the back of the skull, extending onto the shoulders. It is ~15% body surface area, sparing the face and anterior neck. Which test should be ordered?

1 A head CT

2 A head MRI

3 A skull plain film X-ray

4 A skin biopsy

58

Page 59: Skin Tumor MCQ

5 A PET scan

Q/Q(M)-480184 Report a Problem

A child presents with a giant congenital nevus overlying the back of the skull, extending onto the shoulders. It is ~15% body surface area, sparing the face and anterior neck. Which test should be ordered?

2 A head MRI

Giant congenital nevi overlying the spinal columna nd skull can be associated with neurocutaneous melanosis. Presenting symptoms include increased cranial pressure, spinal cord compression or leptomeningeal melanoma. A MRI should be performed in these children to rule-out CNS involvement. The other tests would not be as useful in this situation.

Q/Q(M)-480184 Report a Problem

Which of the following is true regarding actinic keratoses?

1 Salicylic acid, tretinoin, and alpha-hydroxy acids are not useful in treatment

2 Low fat diets may decrease the incidence of AKs

3 Cure rates for treatment with topical 5-fluorouracil are higher than for treatment with cryotherapy

4 UVA from sunlight is most responsible for AK development

5 Male gender is not a risk factor for AKs.

Which of the following is true regarding actinic keratoses?

2 Low fat diets may decrease the incidence of AKs

Salicylic acid, tretinoin, and alpha-hydroxy acids have been reported to be useful in treatment of AKs. Low fat diets have been reported to be useful in decreasing the incidence of AKs. Cryotherapy has a reported cure rate of 98.8% for AKs, whereas topical 5-FU has a reported cure rate of 93%. UVB, not UVA, is most responsible for AK development. Male gender is a risk factor for the development of AKs.

What is the most common location for pagetoid reticulosis?

1 Trunk

2 Head and neck

3 Hands and feet

4 Flexural sites of upper and lower extremities

5 Genitals

What is the most common location for pagetoid reticulosis?

3 Hands and feet

Pagetoid reticulosis is an indolent cutaneous T-cell lymphoma. Pagetoid reticulosis favors an acral distribution and typically presents as scaly oval plaques. Epidermotropism is present, with tumor cells being CD4 positive or CD8 positive. Another interesting immunohistochemical finding is the absence of CD45 expression.

At what location is this tumor LEAST likely to recur after surgical excision?

1 Nasolabial fold

59

Page 60: Skin Tumor MCQ

2 Lateral canthus

3 Mid forehead

4 Preauricular

5 Scalp

At what location is this tumor LEAST likely to recur after surgical excision?

3 Mid forehead

Basal cell carcinomas are slow-growing, epithelial tumors. Highest area of recurrences after surgical excision is in the "H-zone" of the face which include nose, nasolabial folds, periocular, periauricular and the scalp.Which of the following is true regarding digital HPV-associated squamous cell cancers?

1 The rate of metastasis approaches 15%.

2 HPV18 is the most common associated sybtype.

3 Mohs micrographic surgery yields a 20% recurrence rate.

4 Women outnumber men 2:1.

5 These lesions only occur in association with immunosuppression.

Which of the following is true regarding digital HPV-associated squamous cell cancers?

3 Mohs micrographic surgery yields a 20% recurrence rate.

According to Riddel et al (JAAD 2011;64(6):1147-1153), Mohs micrographic surgery, although the treatment of choice, results in a 20% recurrence rate, which is significantly higher than cutaneous SCC. HPV16 is most often implicated. Men outnumber women 2:1. The rate of metastasis averages between 2-3%. Although common in transplant patients, HPV associated digital SCCs can occur secondary to trauma and in immunocompetent patients.Histopathological evidence of epidermotropism may be seen in which of the following conditions:

1 Merkel cell carcinoma

2 SCC

3 CTCL

4 None of these answers are correct

5 All of these answers are correct

Histopathological evidence of epidermotropism may be seen in which of the following conditions:

3 CTCL

Epidermotropism, defined as the presence of lymphocytes in the epidermis is characteristic of CTCL.Merkel cell carcinoma should be treated with what size surgical margins?

1 2mm

2 5mm

3 1cm

4 3cm

5 5cm

Merkel cell carcinoma should be treated with what size surgical margins?

4 3cm

Merkel cell carcinoma is an aggresive rare tumor of the skin accounting for less than 1% of cutaneous malignancies. Also known as neuroendocrine cancer of the skin, this tumor presents as a painless red to violaceous, firm, solitary, nodule that usually presents on sun exposed areas such as the head, neck and upper extremities. These tumors present ussualy during the 6th and 7th decades, andhave a 2 year survival rate of 50-70%. Becuase of this tumors high potenital for regional and distal metastasis, thistumor should

60

Page 61: Skin Tumor MCQ

be excised with wide local excision with 3cm surgical margins.The Grzybowski type of keratoacanthoma:

1 Is characterized by rapid growth of a single lesion reaching a diameter of 9 cm or more

2 Typically invades underlying cartilage

3 Demonstrates simultaneous central healing

4 Presents in childhood on sun-exposed surfaces

5 Presents with hundreds of disseminated lesions

The Grzybowski type of keratoacanthoma:

5 Presents with hundreds of disseminated lesions

The Grzybowski type of keratoacanthoma is typically diagnosed in adulthood, with the sudden appearance of hundreds of lesions in a disseminated fashion. The lesions are generally 2-3mm in diameter and can be found anywhere on the body including the palms, soles, larynx, and oral mucosa.Commonly used immunohistochemical markers that are positive in Merkel cell carcinomas include all of the following EXCEPT:

1 Chromogranin A/B

2 Synaptophysin

3 Glial fibrillary acidic protein

4 Cytokeratin 20

5 Cytokeratin 8

Commonly used immunohistochemical markers that are positive in Merkel cell carcinomas include all of the following EXCEPT:

3 Glial fibrillary acidic protein

The most commonly used markers for MCC are monoclonal antibodies to cytokeratins 8, 18, and 20; neuron-specific enolase (the most constant marker); chromogranin A/B, and synaptophysin. Leukocyte common antigen, vimentin, desmin, glial fibrillary acidic protein, and S-100 are consistently absent in MCC.Topical 5-Fluorouracil:

1 Interferes with the synthesis of DNA and RNA

2 Is an alternative for the treatment of actinic keratosis

3 May cause pruritus and burning at the site of application

4 All of these answers are correct

5 None of these answers are correct

Topical 5-Fluorouracil:

4 All of these answers are correct

Topical 5-FU blocks the methylation reaction of deoxyuridylic acid to thymidylic acid, thus interfering with the synthesis of DNA and RNA. Normal side effects during treatment include pruritus and burning at the site of application. It is a treatment option for patients with multiple AKs within an area.Which of the following is most likely to present with cutaneous metastases in men?

1 Lung cancer

2 Colon cancer

3 Prostate cancer

4 Melanoma

5 Esophageal Cancer

Which of the following is most likely to present with cutaneous metastases in men?

61

Page 62: Skin Tumor MCQ

4 Melanoma

The most common malignancy to present with cutaneous metastases in men is from melanoma, followed by squamous cell cancer of the head and neck. In women, breast cancer is most likely to present with cutaneous metastases. Special presentations of cutaneous metastases include alopecia neoplastica (scarring alopecia) and zosteriform metastases.The mucin found in this lesion is produced by:

1 Fibroblasts

2 Keratinocyte

3 Merkel cells

4 Nail matrix

5 Osteoblast

1 Fibroblasts

Digital mucous cysts are pseudocysts found on the dorsal digits between the distal interphalangeal joint and the nail fold. The fibroblasts in digital mucous cysts produce large quantities of hyaluronic acid.Sentinel lymph node biopsy in malignant melanoma:

1 Has gained acceptance for the treatment of MM of intermediate thickness (1-4mm)

2 Is mainly being used for a more accurate staging

3 Is not recommended

4 None of these answers are correct

5Has gained acceptance for the treatment of MM of intermediate thickness (1-4mm) and is mainly being used for a more accurate staging

Q/Q(M)-474441 Report a Problem

Sentinel lymph node biopsy in malignant melanoma:

5Has gained acceptance for the treatment of MM of intermediate thickness (1-4mm) and is mainly being used for a more accurate staging

Sentinel lymph node biopsy has gained acceptance for the treatment of melanomas 1-4 mm in thickness. Even though it has been widely accepted due to its low morbidity and high feasibility, the role of SLN biopsy in survival has not been established. It is mainly being used for a more accurate staging, prognosis and to determine if further adjuvant therapy is necessary.

Q/Q(M)-474441 Report a Problem

An elderly white man presents with a slowly enlarging, well-demarcated pink, scaly plaque on the neck. Biopsy of the lesion reveals epidermal dysplasia and keratinocytic disorganization with preservation of the basement membrane. Hyperkeratosis and parakeratosis are also present, and numerous atypical keratinocytes are seen throughout the epidermis, with loss of polarity, atypia, and mitoses. A chronic inflammatory infiltrate is present in the upper dermis. Which of the following treatment(s) should be considered for this patient?

62

Page 63: Skin Tumor MCQ

1 Conventional excision

2 Mohs micrographic surgery

3 Imiquimod

4 Conventional excision and Mohs micrographic surgery

5 All of these answers are correct

Q/Q(M)-476479 Report a Problem

An elderly white man presents with a slowly enlarging, well-demarcated pink, scaly plaque on the neck. Biopsy of the lesion reveals epidermal dysplasia and keratinocytic disorganization with preservation of the basement membrane. Hyperkeratosis and parakeratosis are also present, and numerous atypical keratinocytes are seen throughout the epidermis, with loss of polarity, atypia, and mitoses. A chronic inflammatory infiltrate is present in the upper dermis. Which of the following treatment(s) should be considered for this patient?

5 All of these answers are correct

The patient has Bowen's Disease, or squamous cell carcinoma in situ. Multiple treatment options are available, including conventional excision, Mohs micrographic surgery, imiquimod, cryosurgery, and 5-FU, among others. Conventional excision is the most commonly used method. With this method, not only is the lesion removed, but it also provides the specimen for histologic verification to rule out invasive SCC. Recurrence rates of 5% have been reported with conventional excision. Mohs micrographic surgery is an option when the lesion is located in areas where there is an increased risk of sub-clinical spreading, or when tissue sparing is a priority. Imiquimod has recently been successfully used for the treatment of SCCIS, though standardized regimens have not been established.

Q/Q(M)-476479 Report a Problem

Merkel cell carcinoma has been found to be associated with which of the following viruses?

1 Herpes virus

2 Polyomavirus

3 Paramyxovirus

4 Flavivirus

5 Enterovirus

Q/Q(M)-482256 Report a Problem

Merkel cell carcinoma has been found to be associated with which of the following viruses?

2 Polyomavirus

Merkel cell carcinoma has been found to be associated with the merkel cell polyomavirus (MCPyV). Herpes virus

63

Page 64: Skin Tumor MCQ

causes diseases such as HSV, VZV, EBV, CMV, roseola, and Kaposi's sarcoma. Paramyxovirus causes measles. Flavivirus causes diseases such as west nile virus, dengue, and yellow fever. Enteroviruses are not associated with merkel cell carcinoma.

Q/Q(M)-482256 Report a Problem

Which of the following are the most reliable prognostic factors in malignant melanoma?

1 Breslow�s depth and ulceration

2 Breslow�s depth and Clark level

3 Clark level and ulceration

4 Breslow�s depth and Clark level

5 None of these answers are correct

Q/Q(M)-474442 Report a Problem

Which of the following are the most reliable prognostic factors in malignant melanoma?

1 Breslow�s depth and ulceration

The most reliable prognostic factors in MM are Breslow�s depth and ulceration of the primary tumor. Breslow depth is the thickness of the melanoma measured from the granular layer to the deepest point of tumor invasion.

Q/Q(M)-474442 Report a Problem

Seventy-five year old, male patient, with a 3cms pink, pearly nodule on his chest. You notice that the patient has a scar on the same area. The treatment of choice is:

1 Radiation

2 Conventional excision

3 Cryotherapy

4 Mohs surgery

5 Imiquimod

Q/Q(M)-475384 Report a Problem

Seventy-five year old, male patient, with a 3cms pink, pearly nodule on his chest. You notice that the patient has a scar on the same area. The treatment of choice is:

4 Mohs surgery

64

Page 65: Skin Tumor MCQ

The most likely diagnosis in this patient is recurrent BCC, therefore Mohs surgery is the treatment of choice. Furthermore, any NMSC measuring 3 cms located on any anatomical site is an indication for Mohs surgery.

Q/Q(M)-475384 Report a Problem

The risk of a melanoma developing in a giant congenital melanocytic nevus is approximately:

1 3%

2 6%

3 12%

4 50%

5 Virtually all of these patients will develop melanoma

Q/Q(M)-480183 Report a Problem

The risk of a melanoma developing in a giant congenital melanocytic nevus is approximately:

2 6%

Large/giant congenital nevi are greater than 20 cm or greater than 10% of the body surface area. The risk of melanoma in this type of lesion is approximately 6% by the age of 60 (Rhodes, 1981; Bett, 2005). 50% of these melanomas occur by 5 years of age.

Q/Q(M)-480183 Report a Problem

On histologic examination of the adenocarcinoma of the perineal area, which of the following stains would NOT be positive?

1 PAS

2 Mucicarmine

3 CEA

4 EMA

5 HMB45

Q/Q(M)-476848 Report a Problem

65

Page 66: Skin Tumor MCQ

On histologic examination of the adenocarcinoma of the perineal area, which of the following stains would NOT be positive?

5 HMB45

Extramammary Paget's cutaneous adenocarcinoma of glandular differentiation. Approximately 25% of these tumors are associated underlying neoplasms. Histologically, the Paget cells stain positively with PAS, mucicarmine, CEA, EMA, LMW keratin. They are HMB45 negative which is a melanocytic marker.

Q/Q(M)-476848 Report a Problem

Which of the following is not considered a high-risk location of SCC:

1 Eyelids

2 Nose

3 Ear

4 Lips

5 Chest

Q/Q(M)-474447 Report a Problem

Which of the following is not considered a high-risk location of SCC:

5 Chest

High risk locations for recurrence and metastases include the �H-zone� of the face, skin overlying cartilage and bony structures (e.g. preauricular area, retroauricular sulcus, nasolabial fold, inner canthus, philtrum, temple, upper lip, columella, nose, lower eyelid). Lesions located on the trunk and extremities are usually considered low risk unless aggressive histologic pattern is present.

Q/Q(M)-474447 Report a Problem

What virus is most closely associated with these lesions in this HIV infected patient?

1 Human herpes virus 2

2 Human herpes virus 6

66

Page 67: Skin Tumor MCQ

3 Human herpes virus 8

4 Cytomegalovirus

5 Ebstein Barr virus

Q/Q(M)-476684 Report a Problem

What virus is most closely associated with these lesions in this HIV infected patient?

3 Human herpes virus 8

Kaposi's sarcoma is an AIDS defining illness. It if found at a much higher incidence in homosexual men. Human herpes virus 8 is thought to be pathogenic in this tumor.

Q/Q(M)-476684 Report a Problem

Bazex syndrome can be differentiated clinically from Rombo syndrome by presence of

1 Multiple basal cell carcinomas

2 Trichiepitheliomas

3 Milia

4 Bollicular atrophoderma

5 Hypohidrosis

Q/Q(M)-482115 Report a Problem

Bazex syndrome can be differentiated clinically from Rombo syndrome by presence of

5 Hypohidrosis

67

Page 68: Skin Tumor MCQ

Localized hypohidrosis is a feature found in Bazex syndrome but not Rombo syndrome. Another differentiating feature is that Rombo syndrome classically has vermiculate atrophoderma, while Bazex has follicular atrophoderma.

Q/Q(M)-482115 Report a Problem

Which site of squamous cell carcinoma has the greatest risk of metastasis?

1 lip

2 ear

3 eyelid

4 trunk

5 nose

Q/Q(M)-482907 Report a Problem

Which site of squamous cell carcinoma has the greatest risk of metastasis?

1 Lip

The lip has an approximate rate of metastasis of 13%, which is slightly higher than the metastasis rate of 11% from the ear. Risks for metastasis include size greater than 2cm, perineural invasion, immunosuppression, treatment history, degree of tumor differentiation, or location on the ear or lip.

Q/Q(M)-482907 Report a Problem

All of the following are true of melanoma-associated leukoderma except:

1 Lesions resemble vitiligo

2 Portends a worse prognosis

3 Seen in patients with metastatic disease but no primary lesion

4 Histology resembles that of a halo nevus

5 Lesions develop distant to melanoma

Q/Q(M)-477354 Report a Problem

All of the following are true of melanoma-associated leukoderma except:

2 Portends a worse prognosis

Hypomelanosis associated with melanoma most commonly may be seen in three ways. The first type is analogous

68

Page 69: Skin Tumor MCQ

to a halo nevus. The second is a remote leukoderma distant from the primary lesion. The third is a vitiligo like leukoderma. It may portend a better prognosis in comparison to others with the same stage of disease.

Q/Q(M)-477354 Report a Problem

Which of the following is true regarding poikilodermatous mycosis fungoides?

1 Majority of cases are predominantly CD8(+)

2 Patients typically have a later age of onset compared to classic mycosis fungiodes

3 Can be associated with LyP

4 More women than men affected

5 Patients typically do not respond well to phototherapy

Q/Q(M)-482499 Report a Problem

Which of the following is true regarding poikilodermatous mycosis fungoides?

3 Can be associated with LyP

While there is an overrepresentation of CD8+ cases compared to classic mycosis fungoides, more cases are still predominantly CD4+. Patients typically present at a younger age (median age 44 years), with a slight male predominance. There is an excellent response to phototherapy with clinical improvement in ~90% of patients treated. There is an increased association with LyP compared to other types of mycosis fungoides. (Abbott et al, JAAD 2011; in press)

Q/Q(M)-482499 Report a Problem

A 65 year-old female with multiple actinic keratosis on the face under treatment with 5-FU. According to the image and aforementioned information, you may conclude that:

1 The patient must immediately stop treatment since unexpected side effects have developed

2The patient has been compliant with 5-FU treatment and the appearance of inflammation, erythema and erosions are expected

3 The image is not relevant to 5-FU treatment

4 None of these answers are correct

5 All of these answers are correct

Q/Q(M)-474904 Report a Problem

69

Page 70: Skin Tumor MCQ

A 65 year-old female with multiple actinic keratosis on the face under treatment with 5-FU. According to the image and aforementioned information, you may conclude that:

2The patient has been compliant with 5-FU treatment and the appearance of inflammation, erythema and erosions are expected

Compliance is a key feature in treatment with 5-FU. Erythema, inflammation and erosion must develop and is considered a sign of successful treatment.

Q/Q(M)-474904 Report a Problem

The cure rate of cryotherapy as a treatment for actinic keratoses is:

1 80%

2 85%

3 90%

4 95%

5 99%

Q/Q(M)-476108 Report a Problem

The cure rate of cryotherapy as a treatment for actinic keratoses is:

5 99%

Cryotherapy is the most common treatment for AKs, with a cure rate of 98.8%.

Q/Q(M)-476108 Report a Problem

Keratoacanthomas have been linked etiologically to:

1 Ultraviolet exposure

70

Page 71: Skin Tumor MCQ

2 Human papilloma virus

3 Chemical carcinogens such as tar and pitch

4 Smoking

5 All of these answers are correct

Q/Q(M)-476100 Report a Problem

Keratoacanthomas have been linked etiologically to:

5 All of these answers are correct

The origin of KAs has not been established. Ultraviolet exposure, exposure to chemical carcinogens such as tar and pitch, as well as smoking, and a viral etiology, specifically the human papilloma virus, have all been proposed as etiologic factors.

Q/Q(M)-476100 Report a Problem

71